Sunteți pe pagina 1din 261

A 25-year-old female presented with six months

history of depression, irritability and painful sensory


symptoms in her legs. Over the last four weeks she
presents a broad base ataxic gait.
An MRI brain showed bilateral posterior thalamic
nuclei (pulvinar region) high signals.
Which of the following is the most likely diagnosis?
Herpes simplex encephalitis.
Multiple system atrophy.
New variant CJD.
Sporadic CJD .
Wilson disease

New variant Creutzfeldt-Jakob disease (CJD) usually


presents in a young person, in their twenties or thirties.
In the majority of the cases the first symptoms are
psychiatric and painful sensory symptoms in the lower
limbs.
Ataxia and involuntary movements (for example,
myoclonus) usually appear at an interval of about six
months after the initial symptoms.
MRI brain shows bilateral pulvinar (posterior thalamic
nuclei) high signals.
EEG is usually normal in new variant CJD.

A 35 year old woman has been admitted to hospital for


investigation of progressive weakness in her legs. For the
past 5 years. The patient's mother has similar difficulties with
weakness and sensory problems. Examination revealed
power of 3/5 distally in the upper and lower limbs with a glove
and stocking pattern sensory loss to pain and touch.
What is the most likely diagnosis?
A. subacute combined degeneration of spinal cord.
B. Friedrich's ataxia
C. Chronic inflammatory demyelinating polyneuropathy .
D. Multiple sclerosis .
E. Hereditary sensory & motor neuropathy.

Answer: e) hereditary sensori motor neuropathy. In view of the family history, this
patient is most likely to have hereditary sensori motor neuropathy type I (Charcot
Marie Tooth disease). HMSN 1 is the most common form of hereditary neuropathy.
Severely and uniformly slowed nerve conduction velocities (NCVs) and primary
hypertrophic myelin pathology with prominent onion bulbs and secondary axonal
changes are the hallmarks of the disease. Motor symptoms predominate over
sensory symptoms. Often, patients report loss of balance, muscle weakness, and foot
deformities. Onset in the first decade of life is typical, but disease develops in some
patients in young or mid adulthood.
HMSN 2, on the other hand, represents the nondemyelinating neuronal type with
relatively normal NCVs and primary axonal pathology. Although nerves are not
enlarged in the neuronal form, weakness often is less marked and onset of this
neuropathy is delayed. Peripheral nerves are not enlarged clinically, and weakness of
feet and leg muscles predominates; hands are less severely affected than the legs.
Patients experience sensory loss in the distal extremities, and foot deformities (ie,
pes cavus) tend to be less marked than those of HMSN 1.

A 40 year old man presents with an


uncomfortable sensation in his face, comes
to the casualty department. He is unable to
lift his eyebrows and also has bilateral facial
weakness. Which one of the following is
most likely to cause bilateral lower motor
neuron weakness?
A. Cerebrovascular disease.
B. Pontine haemorrhage.
C. Lyme disease.
D. Multiple sclerosis.
E. Motor neuron disease

Answer: c) lyme disease.


Causes of Upper Motor Neuron facial weakness
are:
CVA
pontine haemorrhage
Multiple Sclerosis
motor neuron disease
Lyme disease usually causes lower motor
neuron facial weakness.

A 47-year-old personal assistant attends the outpatient


clinic with a six-month history of malaise and joint pains.
She has had to quit her job. She is also complaining of
difficulty sleeping at night and urinary frequency. The
investigations show: full blood count normal, U&E
normal, LFTs normal, ESR normal, CRP normal, Thyroid
function normal Autoimmune screen normal What is the
most likely diagnosis?
1. fibromyalgia
2. Depression
3. rheumatoid arthritis
4. Vasculitis
5. systemic lupus erythematosus

True 1. Fibromyalgia is said to occur in


between 1 and 2% of the general population
and has a female to male ratio of 9:1,
presenting most often in the 3050 year age
group. It presents with musculoskeletal pain
that often has multiple trigger points and
may be poorly defined. Self-management
strategies, reassurance and tricyclic anti
depressants for sleep disturbance are the
mainstays of therapy. Aerobic exercises such
as swimming are recommended. Symptoms
wax and wane for many years.

33- A teenage girl presents with Guillain-Barre


syndrome. Her weakness continues to worsen
after admission to hospital. Which of the
following should be used to monitor her?
1) arterial blood gases
2) chest expansion size
3) FEV1/FVC ratio
4) PEFR
5) vital capacity

A 46-year-old male presents passing 4-5 litres


of urine per day, after commencing a new drug.
S.sodium 142 mmol/l, Plasma osmolality 295
mosmol/l (275-290), Urine osmolality 280
mosmol/l (350-1000).What drug was
prescribed?
1 )Carbamazepine
2 )Chlorpropamide.
3 )Fluoxetine .
4 )Furosemide .
5 )Lithium.

A 21-year-old Nigerian woman was referred by her general


practitioner with a progressive history of polydipsia and polyuria
of 6 months duration. She has a history of sickle cell disease
and had been admitted on two previous occasions to hospital
with chest pain precipitated by crisis. Fasting blood glucose is
4.5 mmol/l. She is not on any medication.
A water deprivation test was performed. Water deprivation
phase plasma osmolality298 mosm/kg (278300 mosm/kg)
urine osmolality300 mosm/kg
DDAVP Phase plasma osmolality295 mosm/kg,urine
osmolality325 mosm/kg. What diagnosis would best fit the
clinical picture and investigation result?
1. Sickle cell nephropathy
2. Cranial diabetes insipidus
3. Psychogenic polydipsia
4. Nephrogenic diabetes insipidus
5. Lithium-induced nephrotoxicity

True 4. The disease is characterised by the inability to concentrate urine


due to the lack of response of renal tubules to antidiuretic hormone
(ADH). Normally the kidneys concentrate urine by increasing water
resorption by the collecting duct in the presence of ADH; this
mechanism helps to maintain plasma osmolality and extracellular
volume. Without ADH, a large amount of dilute urine is excreted.
The water deprivation test after an overnight fast assesses the
kidneys urine-concentrating ability and response to ADH. The result
shows abnormally low (< 750 mosm/kg) urine osmolality, which
increases only slightly after exogenous ADH (1-deamino-8-D-argininevasopressin (DDAVP), vasopressin). Nephrogenic diabetes insipidus
(NDI) is X-linked recessive. Homozygous affected people (all males)
are completely unresponsive to ADH while heterozygous females
show normal or slight impairment. Acquired NDI occurs in disorders
that disrupt the medulla or distal nephron and impair concentrating
ability. Causes are: sickle cell nephropathy, polycystic kidney disease,
pyelonephritis, amyloidosis, certain nephrotoxins such as lithium and
demeclocycline.

27- In Psoriatic arthropathy all true except


A is usually associated with psoriatic nail
change.
B may cause sacro-iliitis .
C can occurs in the absence of psoriasis of the
skin.
D is associated with uveitis.
E responds to chloroquine, which is the
treatment of choice.

A 25 year-old lady recently diagnosed with rheumatoid


arthritis. She has developed weakness, double vision
and tiredness. Examination reveals bilateral weakness of
eye abduction, bilateral ptosis, slightly reduced proximal
motor power in the limbs, normal reflexes and sensation.
What is the diagnosis?
1) Chronic progressive external opthalmoplegia.
2) Guillain-Barre syndrome.
3) Multiple sclerosis.
4) Myasthenia gravis.
5) Polymyositis

The answer is 4
Myasthenia gravis is well known to be
associated with other autoimmune
diseases such as pernicious anaemia,
thyroid disease and rheumatoid arthritis. In
Guillain-Barre syndrome you will expect
absent reflexes. Polymyositis does not
usually cause ptosis or ophthalmoplegia.

A 70 year man has been short of breath for 1 year. An


electrocardiogram shows T wave inversion and q
waves in the anterolateral leads at rest. He has
cardiomegaly on the chest X ray. Clinical examination
shows a third heart sound, a soft systolic murmur in
the mitral area and also bilateral inspiratory
crepitations in his lungs.
Which of the following is he at risk of?
A. Deep vein thrombosis
B. Pulmonary embolus
C. Systemic arterial embolus from mural thrombus
D. Venous thrombosis due to mural thrombus
E. Coronary artery thrombus due to mural
thrombus

Answer: c) systemic arterial embolus from


mural thrombus. This man has features
indicating that he has an enlarged left
ventricle from previous anterior myocardial
infarction in the LAD artery territory. Poor
LV function also causes his symptoms of
heart failure. He is at risk of developing
mural thrombus with embolus to the arterial
circulation.

A 12 year old girl has a diagnosis of the 21


hydroxylase deficiency form of congenital
adrenal hyperplasia. Which one of these
features is likely to be present?
A. High cortisol .
B. Ambiguous genitalia .
C. Alopecia .
D. Precocious puberty.
E. Hypertension.

Answer: b) ambiguous genitalia. In congenital


adrenal hyperplasia, hirsutism, ambiguous
genitalia and normal puberty (precocious
puberty in boys) are seen in girls. There is also
cortisol deficiency and replacement with
dexamethasone is appropriate. Hypertension
typically occurs in the rarer forms but not 21
hydroxylase deficiency which is the commonest
form of CAH.

21 Hydroxylase deficiency

A 63 year old male is admitted with acute onset


unsteadiness of gait, dizziness and dysphagia.
Examination revealed a right-sided Horner's syndrome,
nystagmus, loss of pain and temperature sensation on
the left side of the trunk and in the left arm and leg, and
gait ataxia.
What is the most likely diagnosis?
1) leaking posterior communicating artery aneurysm
2) left sided acoustic neuroma
3) posterior inferior cerebellar artery occlusion
4) right sided pontine infarct
5) spontaneous left sided cerebellar haemorrhage

The answer is 3
This is Wallenberg's syndrome/ lateral
medullary syndrome and is due to
occlusion of the posterior inferior
cerebellar artery.

A 50 year old lady has polyuria. Her serum


calcium is 2.85 mmol/l and phosphate is 0.9
mmol/l. Which of the following could cause
this?
A. Vitamin D deficiency
B. Loop diuretics
C. Lithium
D. Diltiazem
E. Bisphosphonates

Answer: c) lithium. Lithium, thiazides (not


loop) diuretics and vitamin A or D
intoxication can cause hypercalcaemia.

.A 45 year old patient presents with proximal


muscle weakness, particularly in the lower
limbs. She has a heliotropic rash around
the eyes and also Gottrons papules.
Which one of the following antibodies is
most strongly associated?
A.La
B. Ro
C. Jo-1
A.D. SCL-70
E. Anti DsDNA

Answer: C) Jo-1. The diagnosis is


dermatomyositis. Anti Jo-1 antibody is
associated with acute onset myositis, particularly
dermatomyositis. The limb girdle or proximal
muscles are most severely affected in both
polymyositis and dermatomyositis.

Gottron's papules

A 35 year old lady has grey pigmentation of her skin and


hypotension. Her early morning cortisol is 45 mol/l
and her sodium is 127 mmol/l. Which of the following is
the best replacement regimen?
A. Hydrocortisone 10mg mane, 5mg mid day and 5
mg evening and also fludrocortisone 100 g mane
B. Hydrocortisone 10mg mane, fludrocortisone 100 g
mane
C. Prednisolone 20 mg mane
D. Hydrocortisone 40mg mane and fludrocortisone 300
g mane
E. Hydrocortisone 20mg mane, 10 mg mid day

Answer: a) hydrocortisone 10mg mane,


5mg mid day and 5 mg evening and also
fludrocortisone 100 g mane. She has
Addison's disease and needs both
glucocorticoid (hydrocortisone) and
mineralocorticoid (fludrocortisone)
replacement. Steroid replacement is
usually given 10/5/5 mg or 10/5 mg,
although this is adjusted with cortisol day
curves. Fludrocortisone 100 g mane
should be adequate.

A patient has been in atrial fibrillation for at least


several months. He is brought in for DC
cardioversion because of troublesome
palpitations and breathlessness. His current
medications are digoxin and warfarin. Following
successful cardioversion, which changes
should be made?
A. Discontinue warfarin and digoxin
B. Continue warfarin but not digoxin till
outpatient review
C. Continue digoxin but not warfarin
D. Continue both digoxin and warfarin
E. Continue warfarin for life

Answer: b) continue warfarin but not digoxin till


outpatient review. For at least 4 weeks following
DC cardioversion, the patient is still at risk of
thromboembolism. If the patient remains in sinus
rhythm after 6-8 weeks then warfarin can be
discontinued unless there is evidence of
paroxysmal AF. Digoxin should be stopped once
sinus rhythm is achieved since its role is only for
rate control. It does not help in maintenance of
sinus rhythm.

A 20-year-old man presents with a history of intermittent fever for 1


month, accompanied by chest pain, which is worse on lying down.
He has recently also noticed pain in the joints of his hands and feet.
He is febrile, with a temperature of 39oC. His pulse is 98/min and
his blood pressure is 110/70 mmHg. The positive findings on
examination are enlarged anterior cervical lymph nodes, pericardial
rub, mild hepatomegaly and synovitis of the proximal
interphalangeal (PIP) and wrist joints. Investigations show the
following:Hb 10.5 g/dl, WBC 20 x 10 9/l, P82%, PLT540 x 109/l,
ESR110 mm/1st hr, CRP 246 mg/l, ALT69 IU/l, ALK-P 246 IU/l,
Ferritin 4567 g/l (normal range 14 200 /l), CXR cardiomegaly, 2D echocardiogram pericardial effusion MSU and blood culture
negative, ANA, RF, ANCA all negative, CT abdomen
hepatosplenomegaly. Which of the following is most likely
diagnosis?
1. SLE
2. Mixed connective tissue disease (MCTD)
3. Undifferentiated connective tissue disease (UCTD)
4. RA
5. Adult-onset Stills disease

True 5. Clinical features of adult-onset Stills disease


include arthralgia or arthritis; a high fever (> 39oC),
especially in the afternoon and evening; fleeting
salmon-pink maculopapular rash which is most
prominent with fever; serositis; anaemia; elevated
acute phase reactants; and evidence of
reticuloendothelial activation. In between the febrile
episodes the patient feels rather well.
The laboratory investigations commonly reveal
leucocytosis, thrombocytosis, raised serum ferritin
and an elevated erythrocyte sedimentation rate and
C-reactive protein. Autoantibodies are negative.

A 70-year-old woman referred by a a breast lump.


She was asymptomatic but her investigations
reveal:Corrected calcium2.72 (2.2 2.6)Phosphate0.80 (0.8-1.4)Alkaline
phosphatase110 U/L (20 - 95)PTH
concentration5.1 pmol/L (0.9-5.4)
What is likely diagnosis?
1 )bony metastases
2 )chronic vitamin D excess
3 )ectopic PTH related peptide (PTHrp) secretion
4 )multiple myeloma
5 )primary hyperparathyroidism

A 19-year-old male presents with concerns regarding his


pubertal development. On examination he 1.8 m tall, thin
little pubic & axillary hair. Both testes are approximately 5
mls in volume (Normal 15mls). No other abnormalities are
encountered. Investigations reveal: LH 3.3 mu/l (3-10) FSH
5.5 mu/l (3-10) Testosterone 5.5 nmol/l (9-30) Which of
following is the most likely diagnosis?
1 )Anorexia nervosa .
2 )Craniopharyngioma .
3 )Kallmann syndrome .
4 )Klinefelters syndrome .
5 )Primary testicular failure.

A 74-year-old man has had increasingly severe, throbbing headaches


for several months, centered on the right. There is a palpable tender
cord-like area over his right temple. His heart rate is regular with no
murmurs, gallops, or rubs. Pulses are equal and full in all
extremities, BP is 110/85 mmHg. A biopsy of this lesion is obtained,
and histologic examination reveals a muscular artery with lumenal
narrowing and medial inflammation with lymphocytes,
macrophages, and occasional giant cells. He improves with a
course of high-dose corticosteroid therapy. Which of the following
laboratory test findings is most likely to be present with this
disease?
1) Anti-double stranded DNA titer of 1:1024
2) Erythrocyte sedimentation rate of 110 mm/hr
3) HDL cholesterol of 0.6 mmol/L
4) pANCA titer of 1:160
5) Rheumatoid factor titer of 80 IU/mL

A 21-year-old man has recently returned from a


holiday in the Far East. He gives a history of
symmetrical joint pain in his knees and ankles
and feet and also complains of sore eyes. His
ESR is raised. A synovial fluid aspirate is sterile
but with a high neutrophil count.What is the
most likely diagnosis?
1. Ankylosing spondylitis
2. Septic arthritis
3. Reactive arthropathy
4. Rheumatoid arthritis
5. Stills disease

True 3. The history of recent travel to the


Far East raises the possibility of
venereal disease. The lower limb
symmetrical arthropathy is
characteristic of reactive arthropathy. A
history should be taken of
urethritis/dysuria and conjunctivitis to
complete the Reiters syndrome triad.

A 32-year-old woman presents a four month history of amenorrhoea.


She takes no specific therapy. She has two children her husband has
a vasectomy. Examination reveals an obese individual but no other
abnormality.
Investigations reveal:Serum oestradiol 100 pmol/L(NR 130 - 500) S.
LH 2.1 mU/L(NR 3.0 - 6.6) S. FSH 2.2 mU/L(NR 3.3 10) S. prolactin
800 mU/L(NR 50 - 300)Serum testosterone2.1 pmol/L(NR < 3.0)
Which investigation
1 )Insulin tolerance test
2 )Pregnancy test
3 )17 hydroxy-progesterone
4 )Urine free cortisol concentration
5 )Magnetic resonance imaging (MRI) of pituitary .

A 75-year-old man presents with 12 months history of


cognitive impairment, parkinsonism, intermittent
confusion and generalised myoclonus. He was started
on 62.5 tds of sinemet. In the following 2 months he was
started experiencing visual hallucinations. The most
likely diagnosis is:
1) Idiopathic Parkinson's disease
2) Alzheimer's disease
3) Diffuse Lewy body disease
4) Multiple system atrophy
5) Progressive supranuclear palsy

A 70-year-old man from Lancashire has noted increasing


back & leg pain over several years. X-rays reveal bony
sclerosis of the sacroiliac, lower vertebral, & upper tibial
regions with cortical thickening, but without mass effect
or significant bony destruction. He also says his hat
does not fit him anymore. He has greater difficulty
hearing on the left. He has orthopnea & pedal edema.
Blood tests reveal an elevated serum alkaline
phosphatase. The most likely pathologic process that
explains these findings is?
1 )Decreased bone mass
2 )Metastatic adenocarcinoma
3 )Paget's disease of bone
4 )Renal failure & renal osteodystrophy
5 )Vitamin D deficiency

Comments: true 3.
This man has Paget's disease with high output
cardiac failure & sensorineural deafness.
Renal osteodystrophy leads to lesions of osteitis
fibrosa cystica admixed with osteomalacia,
which are focal in nature. Metastatic disease in
bone produces focal lesions.

A 17 year presents tingling & muscle cramps. There


is no other past history of note. Investigations reveal
Creatinine68 micromol/L (50-100)calcium1.76 (2.22.6)albumin38 g/L (37-49)
Which one of following investigations is likely to
confirm diagnosis?
1 )Alkaline phosphatase concentration
2 )CT brain scanning
3 )PTH concentration
4 )Urine calcium concentration
5 )Vitamin D concentration

An otherwise healthy middle-aged man without


prior medical history has had increasing back
pain & right hip pain over the past 10 years.
The pain is worse at the end of the day. He
has bony enlargement of the distal
interphalangeal joints. A radiograph of the spine
reveals the presence of prominent osteophytes
involving vertebral bodies. There is sclerosis
& narrowing of the joint space at the right
acetabulum seen on a radiograph of the pelvis.
Which of the following pathologic processes is
likely to be taking place in this patient?
1 )Gout
2 )Lyme disease
3 )Osteoarthritis
4 )Osteomyelitis
5 )Rheumatoid arthritis

Comments: true 3.
Degenerative osteoarthritis is a common & progressive
condition that becomes more frequent symptomatic
with aging. There is erosion & loss of articular
cartilage. Rheumatoid arthritis typically involves small
joints of the hands & feet, there is a destructive
pannus that leads to marked joint deformity. A gouty
arthritis is more likely to be accompanied by swelling,
deformity & joint destruction. The pain is related to
usage. Osteomyelitis represents an ongoing infection
that produces marked bone deformity, not just joint
narrowing. Lyme disease produces a chronic arthritis,
but it is typically preceded by a deer tick bite & a skin
lesion. It is much less common than osteoarthritis.

A 62-year-old man attends for review. He has


evidence of arthritis affecting the shoulder, elbow,
radiocarpal, and knee joints bilaterally, with pain
also in the left first metatarso-phalangeal joint. Xray of the left knee reveals calcification within the
hyaline cartilage. Aspiration of the left knee
reveals calcium pyrophosphate crystals. Serum
uric acid is slightly above the upper limit of normal.
Other medical history of note is ulcerative colitis.
What is the most likely diagnosis in this case?
1. Gout
2. Pseudogout
3. Primary osteoarthritis
4. Rheumatoid arthritis
5. Sero-negative arthritis

True 2. The clinical picture of joint involvement, coupled with


evidence of chondrocalcinosis and pyrophosphate crystals is
strongly suggestive of a diagnosis of pseudogout. The slightly
elevated uric acid and history of ulcerative colitis are redherrings in this case. Management involves use of simple
painkillers and non-steroidal anti-inflammatory agents coupled
with physiotherapy where appropriate. Associated conditions
include haemochromatosis and Wilsons disease, where there
are other symptoms and signs it may be worth screening for
these concomitant illnesses. Pseudogout has an equal
male:female ratio, with frequency increasing with increasing
age; it is said to affect around 5% of subjects above the age of
30 years.

35- A 35-year-old man returned from a two-week holiday


complaining of pain in the loins and painful swollen knees. On
examination he was afebrile and had significant bilateral knee
effusions. Mild penile erythema was also noted. Laboratory
investigations showed.
Hb 15.6 g/dL
WBC 16.2 x 109/l
Neutrophils 14.1 x 109/l
ESR 65 mm/h
Rheumatoid factor 10 IU/L
Urinalysis No cells, casts or bacteria seen

What is the most likely diagnosis?


1) Arthritis due to Neisseria gonorrhoeae infection.
2) Lymphogranuloma venereum.
3) Reactive arthritis.
4) Reitter's syndrome.
5) Rheumatoid arthritis.

A 70-year-old man presents with weight loss, lower limb


weakness and dry mouth. He has been a heavy smoker.
On examination, he looks cachectic; he has proximal
lower limb weakness, areflexia (reflexes normalise with
repetitive muscle contraction). There is no wasting or
fasciculations. Sensory examination is normal. Which of
the following blood test is the most likely to confirm the
diagnosis?
1) Acetylcholine receptors
2) Voltage gated calcium channels antibodies
3) Anti GM1 antibody
4) Antinuclear antibody
5) Anti Ro/La antibodies

The answer is 2
The most likely diagnosis is Lambert-Eaton syndrome. It
results when IgG autoantibodies blockade the voltagegated calcium channels of peripheral cholinergic nerve
territory. 50% of the cases are associated with small cell
lung carcinoma. Proximal lower limb weakness is the
most consistent neurological feature. Ptosis and
ophthalmoplegia are rare. Autonomic dysfunction is
common (e.g. dry mouth). The reflexes are depressed or
absent but normalise with repetitive muscle contraction.

A 42-year-old man reviewed in outpatients has a 6month history of increasing shortness of breath on
exertion and feelings of lightheadedness when
digging in his garden. His general practitioner
organised an open access echocardiogram which
showed a septal thickness of 26mm and a left
ventricular outflow gradient of 85mmHg. Which of
the following is NOT a risk factor for sudden death
in patients with hypertrophic cardiomyopathy?
A : Unexplained syncope
B : Sudden death from HOCM in 2 or more first
degree relatives <40years of age
C : Family history of sudden death
D : left ventricular wall thickness of >30mm
E : Hypertension.

Major risk factors for hypertrophic


cardiomyopathy are:
Cardiac arrest (ventricular fibrillation)
Spontaneous sustained ventricular tachycardia
Family history of sudden death.
Unexplained syncope
Left ventricualr wall thickness>30mm
Abnormal blood pressure on exercise(failure to
rise from baseline by 25mmHg)
Non sustained Ventricular tachycardia

61- 54 year old female is admitted with


progressive weakness following a trivial flulike illness. Which of the following would
exclude Guillain-Barre Syndrome as the
diagnosis?
1) Autonomic dysfunction.
2) Elevated protein on CSF examination.
3) Areflexia.
4) Ophthalmoplegia.
5) Sensory level below D1.

The answer is 5
GBS is a post-infectious acute polyneuritis typified
by elevated CSF protein with few cells and often
normal glucose. There is a profound weakness
associated with areflexia and peripheral sensory
neuropathy. Ophthalmoplegia is associated in
particular with the Miller-Fisher variant. However,
a sensory level is NOT a feature and would
suggest cervical myelopathy. Muscle wasting is
typical with prolonged illness. Autonomic disease
may also feature.

A 68 year-old presents with a 4 month history of weight loss,


headaches & had recently developed double vision. 6 years
previously she underwent a right mastectomy for breast carcinoma
& remains on treatment with Tamoxifen.
Examination shows tenderness over the temporal region & a left
sixth nerve palsy.
Her chest X-ray was reported as normal, but she had an ESR of 100
mm/hr , her Hb was 10.8 g/dl. Which of following statements
correct?
1 )An isotope bone scan should be performed
2 )An urgent CT brain scan required
3 )She should be treated prednisolone immediately
4 )She should have a lumbar puncture
5 )She should be given Diamorphine

An 18 year old man presented with a history of a sudden


onset of a frontal headache and photophobia. He had
neck stiffness and a temperature of 38C.
Which one of the following findings would suggest a
diagnosis of subarachnoid haemorrhage rather than
bacterial meningitis?
1) a blood neutrophil leucocytosis
2) a family history of polycystic renal disease
3) a fluctuating conscious level
4) a history of diabetes mellitus
5) a history of opiate abuse

The answer is 2
Fluctuating level of consiousness can occur
in both meningitis and subarachnoid
haemorrhage (SAH). Hypertension is a
risk factor for SAH, but not diabetes.
Opiate abuse does not increase the risk
for SAH. Cerebral aneurysm are
associated with polystic kidney disease.

An 80-year-old woman has a three month history of progressive


numbness and unsteadiness of her gait. On examination, there is a
mild spastic paraparesis, with brisk knee reflexes, ankle reflexes are
present with reinforcement, extensor plantars, sensory loss in the
legs with a sensory level at T10, impaired joint position sense in the
toes, and loss of vibration sense below the iliac crests.
Investigations were as follows:haemoglobin 12.0 g/dl
MCV 99 fl
What is the most likely diagnosis?
1) anterior spinal artery occlusion
2) dorsal meningioma
3) multiple sclerosis
4) subacute combined degeneration of the cord
5) tabes dorsalis

A 46-year-old male presents passing 4-5 litres


of urine per day, after commencing a new drug.
S.sodium 142 mmol/l, Plasma osmolality 295
mosmol/l (275-290), Urine osmolality 280
mosmol/l (350-1000).What drug was
prescribed?
1 )Carbamazepine
2 )Chlorpropamide.
3 )Fluoxetine .
4 )Furosemide .
5 )Lithium.

A 50-year-old man presented with a six-week history of


general malaise and a 2 day history of a right foot drop,
a left ulnar nerve palsy and a widespread purpuric rash.
He complained of arthralgia but had no clinical evidence
of inflammatory joint disease. Investigations revealed:
ESR100 mm/hr, ANCA negative, ANA negative,
Rheumatoid factor strongly positive, C3 0.8 g/L (NR
0.75 - 1.6), C4 0.02 g/L (NR 0.14 - 0.5), Urine dipstick
Blood ++, no protein. An echocardiogram was normal
and two sets of blood cultures were negative.What is the
most likely diagnosis?
1 )ANA negative SLE
2 )Cryoglobulinaemia
3 )Infective endocarditis
4 )Polyarteritis nodosa
5 )Rheumatoid arthritis

The answer is 2
The presence of a sensory loss at T10
indicates a thoracic mylopathy. Subacute
combined degeneration of the cord is
unlikely as Hb and MCV are normal.
Anterior spinal artery occlusion is unlikely
as the history is progessive.

63- A 45-year-old man has a history of progressive


weakness for 5 weeks. He had particular difficulty getting
out of the bath. On examination there was severe truncal
and proximal limb weakness, without wasting or
fasciculation. Tendon reflexes, plantar and sensation
were all normal. The vital capacity was 1.8L. What is the
most likely diagnosis?
1) cervical myelitis
2) Guillain-Barre syndrome
3) polio
4) polymyositis
5) syringiobulbia

The answer is 4
The presentation of myopathy is
characterised by priximal weakness with
normal reflexes and sensation and the
absence of fasciculations. Polymyositis is
the commonest cause of inflammatory
muscle disease in < 50 years old
(inclusion body myositis is the commonest
in >50 years old).

A 17-year-old male is investigated for short stature. He has a previous


diagnosis of slipped femoral epiphysis diagnosed at the age of 11. He
presented with this disorder at this age with pain in the hip and limp. This
was treated by the orthopaedic surgeons with nonsurgical containment of
the femoral head in the acetabulum using casts.

On examination he is on the 12th centile for height, a BMI of 30 and has


normal pubertal development. His blood pressure is 108/70 mmHg and he
has a pulse of 90 beats per minute. No abnormalities are noted on
examination of the chest, heart and abdomen.
Investigations reveal:
Haemoglobin 12.8 g/dL (13.0-18.0) White cell count 5.4 x109/L (4-11
x109) Platelets 143 x109/L (150-400 x109) Serum Na 133 mmol/L (137144)Serum K 4.2 mmol/L (3.5-4.9) Serum Creatinine 96 mol/L (60-110)
Serum Calcium 2.02 mmol/L (2.2-2.6) Serum Phosphate 1.8 mmol/L (0.81.4) PTH 15.8 pmol/L (0.9-5.4).

Which of the following is the most likely explanation of this boy's presentation?
Coeliac disease.
Primary hypoadrenalism .
Pseudohypoparathyroidism.
Renal osteodystrophy.
Vitamin D resistant osteomalacia.

This young boy has short stature and previous


history of slipped femoral epiphysis.
The investigations reveal a hypocalcaemia and a
hyperphosphataemia suggesting a
hypoparathyroidism (both calcium and
phosphate would be expected to be low in
vitamin D deficiency and hypophosphataemia in
vitamin D resistant rickets) yet the raised
parathyroid hormone (PTH) concentration is
elevated indicating pseudohypoparathyroidism.

A 55-year-old man who had received


haemodialysis for many years presents with
deteriorating discomfort in both shoulders. Past
medical history included bilateral carpal tunnel
decompression. His Investigations reveal:
haemoglobin10 g/dl, ESR 30 mm/1st hr (1-10),
C-reactive protein 12mg/L (1-10), Urate 0.58
(less than 0.45)What is the most likely
diagnosis?
1 )B2 microglobulin amyloidosis
2 )Gout
3 )Pseudogout
4 )Polymyalgia rheumatica
5 )Osteoarthritis

Comments: true 1.
The features of shoulder pain associated with a
past history of carpal tunnel syndrome in a
patient receiving haemodialysis suggests a
diagnosis of b2 microglobulin amyloidosis.
Amyloid deposits composed of b2-microglobulin
as a major constituent protein are mainly
localized in joints with periarticular bone that
lead to destructive arthropathy which tends to
develop 5- 10 years after the initiation of
dialysis. Death from amyloidosis of gut & heart
may occur after 20 years of dialysis.

The dual X-ray bone absorptiometry (DXA)


scan results of a 60-year-old man with RA
being treated with prednisolone 7.5 mg
per day and methotrexate 10 mg per week
show T scores of -1.6 at spine and - 1.8 at
hips. Which of the following would you
add?
1.Calcium and vitamin D
2.Parathyroid hormone
3.Alendronate
4.Strontium
5.No change in therapy

True 3. A T score of less than -2.5 at hip or spine is


indicative of osteoporosis. However, in patients on
steroid therapy, a score of -1.5 is taken as a cut-off
value to start osteoporosis therapy. Use of
bisphosphonates (alendronate and risedronate) has
been shown to be associated with a reduced risk of
fractures. Bisphosphonates are most commonly
used first-line agents for the treatment of
osteoporosis. Calcium and vitamin D alone are not
sufficient in reducing the risk of fractures, although
they do offer some benefit. They may be given
concomitantly with bisphosphonates. Strontium and
parathyroid hormone are used in case of intolerance
or lack of response to bisphosphonates.

A 16 year old girl presented with a three week history of headache and
horizontal diplopia on far right lateral gaze. On two separate
occasions she noted dimmed vision whilst bending forwards. Over
the last year she had gained 12 kilograms in weight. On
examination, her weight was 95 kg, and height 162cms.
Neurological examination revealed bilateral papilloedema and a
partial right sixth cranial nerve palsy. What is the most likely
diagnosis?
1) Benign intracranial hypertension.
2) Multiple sclerosis.
3) Pituitary tumour
4) Superior sagittal vein thrombosis.
5) Thyroid eye disease.

The answer is 1
This patient is markedly obese with a BMI of 36 and the
history suggestive of BIH. Vision may be affected with
enlargement of the blind spot and the visual obscuration
with movements that provoke a rise in ICP (eg bending)
is typical of BIH. Dysthyroid eye disease would not
present like this and is more commonly associated with
Hyperthyroidism. The papilloedema would argue against
MS. A bitemporal hemianopia or a visual field defect
would be expected with a pituitary tumour. Venous sinus
thrombosis is a possibility but would be expected to
produce deteriorating symptoms.

5.A 30-year-old lady with a history of intravenous drug use is


admitted feeling generally unwell. She has a fever and is
complaining of arthralgia and a photosensitive rash. On
examination she has a soft systolic murmur at the lower left
sternal edge.
Investigations reveal: Electrocardiogram Sinus tachycardia,
Haemoglobin10.2 g/dl,White cell count 12.2 109/l,
Platelets,474 109/ l , Sodium 132 mmol/l, Potassium 4.6
mmol/l , Urea15.2 mmol/l,Creatinine 145 mol/l,Urine
microscopy=Red cell casts,Erythrocyte sedimentation rate 32
mm/h (020),C-reactive protein 64 mg/l (<10), Complement
C3--- 20 mg/dl (65190), Complement C4 --- 3 mg/dl (1550),
Renal biopsy=Focal segmental proliferative glomerulonephritis
What are the next appropriate investigations?
1. Antinuclear antibodies.
2. Rheumatoid factor.
3. Echocardiography.
4. Anticardiolipin antibodies.
5. Chest x-ray.

6. One of the following features is


consistent with a diagnosis of
polymyalgia rheumatica:
A
B
C
D
E

proximal muscle pain & stiffness.


an increased creatine kinase activity .
proximal muscle weakness.
an abnormal EMG.
a macrocytic anaemia .

Which is true of herpes simplex


encephalitis?
1) brain MRI is characteristically normal
2) fits are uncommon
3) genital herpes is usually present
4) temporal lobe involvement is common
5) viral identification using polymerase chain
reaction on CSF is non-specific

The answer is 4
Herpes simplex encephalitis (HSE) is associated
with high signal in one or both temporal lobes
(limbic encephalitis). Seizures are commonly
present in HSE. Herpes Simplex Virus type 1 is
the causative virus (Not type 2 which is
associated with genital herpes). PCR for herpes
simplex virus on CSF is highly specific test.

A patient who has rheumatic mitral stenosis is


considered for percutaneous mitral valvuloplasty.
Which of the following would contraindicate this
procedure?
A. Dilated left atrium
B. Atrial fibrillation
C. Aortic regurgitation
D. Heavy calcification of the mitral valve
E. Long history of mitral stenosis

Answer: d) heavy calcification of the mitral


valve. The contraindications towards
valvuloplasty are heavy MV calcification,
thrombus in the left atrial appendage on
transoesophageal echocardiography and
severe mitral regurgitation. These patients
are indicated for mitral valve surgery
instead.

A 35 year old woman is admitted with a blood


pressure of 230/120. She has a sinus tachycardia
of HR 160 with intermittent runs of non sustained
ventricular tachycardia. 24 hour urine shows
increased Adrenaline of 720 (<80 nmol/24 hours)
and Noradrenaline 2300 (<780 nmol/24 hours).
Which one of the following medications would be
most useful?
A. Intravenous labetalol.
B. Intravenous amiodarone.
C. Intravenous phentolamine.
D. Intravenous diltiazem .
E. Oral flecainide.

Answer: c) intravenous phentolamine. In the


management of acute hypertensive crisis of
phaeochromocytoma as in this case, IV administration of
sodium nitroprusside, nitroglycerine, or phentolamine
can be used. Preoperatively, phenoxybenzamine
preoperative adrenergic-blockade of a1 and a2 receptors
with phenoxybenzamine (10-30 mg twice daily), or a1
receptors with prazosin (starting with 1 to 2 mg three
times daily. Beta blockers can be useful for arrhythmias,
but should not be commenced before alpha blockers
because b-blockade alone can cause marked
hypertension.

A 50 year old man presents with breathlessness.


His chest XR shows cardiomegaly. Which of the
following in the history might elucidate a cause
of cardiomyopathy?
A. Inferior T wave inversion
B. Systolic murmur in the mitral area
C. History of diabetes and a tanned
complexion
D. Family history of hyperlipidaemia
E. Family history of myocardial infarction

Answer: c) history of diabetes and a


tanned complexion. A history of diabetes
and bronze / tanned pigmentation suggest
haemochromatosis. Liver function tests
and iron studies would help to confirm the
diagnosis.

A 68 year old man with bone pain has the


following blood results:
calcium 2.27 (2.25-2.7) mmol/l
phosphate 1.3 (0.8-1.45) mmol/L
ALP 335 (20-120) U/l
What is the likely diagnosis?
A. Multiple myeloma
B. Pagets disease
C. Osteoporosis
D. Osteomalacia
E. Primary hyperparathyroidism

Answer: B) Pagets disease In pagets


disease, increased plasma alkaline
phosphatase reflects osteoblastic activity. .
Parathyroid hormone secretion usually
maintains a normal level of calcium ions in
the serum. Concentration of phosphate in
the serum may be normal or slightly
elevated.

A 21 year old female with epilepsy is well controlled on


sodium valproate 600mg bd and had been taking oral
contraceptives for three years. She presented to her
general practitioner 12 weeks pregnant.
Which of the following is correct?
1) An alternative anticonvulsant should be used in place of
sodium valproate
2) Interaction of sodium valproate with the oral
contraceptive increased the risk of pregnancy
3) The dose of sodium valproate should be increased
4) There is an increased risk of a neural tube defect in her
fetus
5) She is at increased risk of anaemia in pregnancy

The answer is 4
There is an increased risk of neural tube defects
associated with anti-convulsants during
pregnancy. However, the risks associated with
treatment are outweighed by the benefits in
preventing seizures, so the drug should be
continued. The risks may be minimised through
use of folate supplements. Sodium valproate is
not an enzyme inducer and would not speed up
metabolism of the pill.

A 35 year old woman is suspected of having


systemic sclerosis following presentation with
Raynaud's phenomenon. Which one of the
following is an associated feature?
A. Lax skin on the hands.
B. Non blanching purpura .
C. Perioral puckering.
D. Liver nodules.
E. Mesothelioma .

Answer: c) perioral tethering. Systemic


sclerosis is usually associated with dry
mouth and dry eyes, raynaud's
phenomenon, telangiectasia and perioral
tethering.

Perioral tethering in systemic sclerosis

A 60 year old woman has had difficulty walking due to


unsteadiness. She has lost 5 kg in weight over the past 6
months. On examination, she has horizontal nystagmus.
Her speech is slurred. There is incoordination of the
upper limbs evident on past pointing. Her gait is ataxic.
General examination reveals a palpable breast lump.
Which of the following is most likely to yield the
diagnosis?
A. Anti Yo antibodies
B. Anti Hu antibodies
C. Anti GAD antibodies
D. MRI of the brain
E. CT of the head

Answer: a) Anti Yo antibodies. This lady


has a cerebellar syndrome which may be
related to a breast malignancy. Anti Yo
antibodies are associated with a cerebellar
syndrome due to either lung, breast or
ovarian carcinoma.

Regarding pseudotumours cerebri (benign


hypercranial hypertension) which is true?
1) A mildly increased CSF cell count is typical.
2) May be caused by prolonged steriod therapy.
3) Is occasionally associated with focal
neurological signs.
4) Frequently presents with ataxia.
5) Is distinguished from hydrocephalus by the
absence of suture separation.

The answer is 2
Pseudotumour cerebri is a clinical syndrome that mimics brain tumours, and is characterised by
raised intracranial pressure with normal CSF cell count and protein content, normal ventricular
size, anatomy and position. Causes:
Metabolic disorders: galactosaemia, hypoparathyroidism, pseudohyperparathyroidism,
hypophosphatasia, steroid therapy, hypervitaminosis A, vitamin A deficiency, Addison's Disease,
obesity, menarche, oral contraceptives, pregnancy.
Infections: Roseola infantum, chronic otitis media, mastoiditis, Guillain Barr Syndrome. Drugs:
Nalidixic acid, tetracycline.
Haematological disorders: Polycythemia, haemolytic and iron deficiency anaemia, Wiskott Aldrich
Syndrome.
Destruction of intracranial drainage by venous thrombosis: Lateral sinus or posterior saggital sinus
thrombosis, head injury, obstruction of the superior vena cava. It usually presents with headache
and vomiting, though this is rarely as bad as that associated with posterior fossa tumour.
Diplopia is common due to 6th nerve palsy. Children are alert with no systemic upset. A bulging
fontanelle, cracked pot sounds, or separation of the cranial sutures may be present.
Papilloedema with an enlarged blind spot is the most consistent sign beyond infancy. Focal and
neurological signs indicate a process other than pseudotumour cerebri. It may be complicated by
optic atrophy and blindness. Most can be treated conservatively with monitoring of visual acuity.
For others, multiple lumbar punctures may be necessary to reduce intracranial pressure. Very
rarely are shunts required.

Which of the following features is


characteristic of myasthenia gravis?
1) Diplopia
2) Equal sex incidence
3) Fasciculation
4) Lid lag
5) Loss of pupillary reflexes

The answer is 1
Myasthenia gravis is commoner in females (it is an
autoimmune disease). The commonest features
include ptosis, diplopia and ophthalmoplegia. It
is a neuromuscular disorder and therefore does
not cause any lower motor neuron signs such as
fasciculations, wasting, and loss of reflexes.
Pupils are always normal. Lid lag is a feature of
thyroid eye disease.

A 68-year-old man presents with progressive


visual impairment. On examination there is an
incongruous homonymous hemianopia. The
most likely anatomical site of the neurological
lesion is at:
1) optic nerve
2) optic tract
3) chiasma
4) optic radiation
5) occipital lobe

The answer is 2
Optic neuropathy causes a central scotoma,
an optic tract lesion an incongruous
homonymous hemianopia, a chiasmal
lesion a bitemporal hemianopia, an optic
radiation and occipital lobe lesion a
congruous homonymous hemianopia.

A 25-year-old female presents with 2 days history of


diplopia and unsteadiness. 2 weeks ago she suffered an
upper respiratory tract infection. On examination there is
complete opthalmoplegia, areflexia and gait ataxia.
Which of the following blood tests is the most likely to
confirm the underlying diagnosis?
1) Acetylcholine receptors antibodies
2) Anti GM1 antibodies
3) Anti GQib antibodies
4) Anti Hu antibodies
5) Anti purkinje cell antibodies

The answer is 3
The most likely diagnosis is Miller Fisher syndrome (variant of Guillain Barre
syndrome). It consists of complete or partial ophthalmoplegia, areflexia and
ataxia. It usually follows antecedent infections. Serum IgG antibody to the
ganglioside GQib is present in more than 95% of patients. It is highly specific
for the syndrome.
Elevated levels of antibodies to the glycolipid ganglioside-monosialic acid (GM1
antibodies) have been shown, in some instances, to be associated with
certain neurological disorders: lower motor neuron syndromes, amyotrophic
lateral sclerosis, multiple sclerosis, other multifocal neuropathies, and
systemic lupus erythematosus (SLE) with central nervous system
involvement.
Neuronal Nuclear (Hu) Antibodies (NNA) are found in a number of
paraneoplastic syndromes, including subacute sensory neuronopathy,
paraneoplastic encephalomyelitis and paraneoplastic cerebellar
degeneration and are associated with small cell lung carcinoma.
Purkinje cell cytoplasmic antibodies are useful for identifying individuals with
subacute cerebellar degeneration or peripheral neuropathy due to a remote
(autoimmune) effect of gynecologic or breast carcinoma.

A 17-year-old man has been diagnosed with schizophrenia


4 weeks ago. He was started on haloperidol. Two weeks
later he was found confused and drowsy. On
examination he was pyrexial (40.7 C), rigid with blood
pressure of 200/100. Which of the following treatment
will you initiate?
1) phenytoin
2) diazepam
3) cefuroxime
4) acyclovir
5) dantrolene

The answer is 5
Neuroleptic malignant syndrome is the most likely
diagnosis. Its major features are: rigidity, altered
mental state, autonomic dysfunction, fever, and
high creatinine kinase. It is usually caused by
potent neuroleptics. The treatment of choice is
dantrolene and bromocriptine. Withdrawal of
neuroleptic treatment is mandatory.
Rhabdomyolysis and acute renal failure are
potential complications.

A 60-year-old woman presents with a short history of sinus


congestion, epistaxis and joint pains in her hands. She
becomes increasingly short of breath and develops
haemoptysis precipitating admission to hospital. On direct
questioning she admits to passing less urine in the preceding
few days. On examination, Heart sounds are normal with no
murmurs. She has bibasilar fine crackles on auscultation of
the lungs but her JVP is not elevated and she has no pedal
oedema. A chest X-ray (CXR) shows bilateral air space
shadowing throughout both lung fields. Urinalysis
demonstrates red blood cells and red blood cell casts. Her
haemoglobin (Hb) is 6 g/dl and her urea is 45 mmol/l, with a
creatinine of 800 mol/. What is the most likely diagnosis?
1. IgA nephropathy
2. Goodpasture's syndrome
3. Antineutrophil cytoplasmic antibody (ANCA)-positive
vasculitis
4. Cryoglobulinemia
5. Uric acid nephropathy

True 3. This is a classic history for Wegener's granulomatosis.


She would need treatment with dialysis; immunosuppression
and most centres would advocate plasma exchange with a
creatinine above 400 mol/l. None of the other diagnoses would
classically cause sinus or joint involvement, and pulmonary
haemorrhage is typically seen only in Goodpasture's syndrome
and ANCA-positive vasculitis. Note (LVF) can lead to pink
sputum and may be confused for pulmonary haemorrhage. To
confirm the diagnosis of pulmonary haemorrhage, a raised
potassium channel opening (K+-CO) would be seen on lung
function tests. The low haemoglobin (Hb) in the setting of acute
renal failure and the disproportionate degree of abnormality
seen on chest X-ray (CXR) compared to clinical findings
suggest pulmonary haemorrhage rather than infection or fluid
to explain the CXR findings.

26-A 25-year-old man presents with right-sided


facial weakness and swelling. There is a right
LMN facial nerve palsy but no other abnormalities
on examination. Lumbar puncture findings are:
Opening pressure18 cm, CSF Protein 0.9 g/l
Glucose3.5 mmol/l, Microscopy 85 lymphocytes.
What is the most likely diagnosis?
A. Lyme disease
B. Multiple sclerosis (MS)
C. Guillain Barre syndrome (GBS)
D. Neurosarcoidosis
E. Ramsey Hunt syndrome

An 81-year-old female presents with bilaterally


painful knees. There was no history of
gastrointestinal diseases. On examination she
had crepitus but had a full range of movement of
both knees. Which one of the following is an
appropriate initial treatment for her painful
knees?
1 )Dihydrocodeine
2 )Naproxen
3 )Paracetamol
4 )Celecoxib
5 )Topical Diclofenac

Comments: true 3.
This woman has osteoarthritis (OA) of both knees. The
principle goal of systemic therapy is to provide an
effective pain relief with the least associated toxicity.
Paracetamol is the initial therapy recommended for the
treatment of OA of the hip & knee. Studies have shown
that short-term & long-term efficacy of paracetamol is
comparable to that of ibuprofen & naproxen in people
with knee osteoarthritis. Specific COX-2 inhibitors such
as celecoxib have clinical benefit similar to that of
traditional NSAIDS, but less GI toxicity although issues
remain regarding their cardiovascualr risk. They may be
used in patients with GI intolerance of traditional
NSAIDs.

A 50 year-old male epileptic presents with paraesthesia of


hands and feet. He also has unsteadiness when walking.
On examination he has a peripheral sensory neuropathy
and palpable lymph nodes in his neck and axillae.
Which of the following drugs is the most likely cause of
these features?
1) Carbamazepine.
2) Clonazepam
3) Lamotrigine.
4) Phenytoin.
5) Sodium valproate.

The answer is 4
Phenytoin is well known to cause
neurological side effects such as
peripheral sensory neuropathy and
cerebellar ataxia. Other side effects
include gingival hypertrophy,
lymphadenopathy hypocalcaemia,
hirsutism.

18. A 45-year-old male attends for an insurance & is


in good health. Examination was normal but investigations
reveal that he has a serum urate concentration of 0.55
mmol/l (NR 0.25-0.45). Which of the following is the most
appropriate management of this patient?
1 )Lifestyle advice.
2 )Start Allopurinol .
3 )Start Colchicine.
4 )Start Diclofenac.
5 )Start Prednisolone.

A 30-year-old woman presented with a deep vein


thrombosis. Her previous history included
investigation for infertility. Investigations
revealed: Haemoglobin 12.8 g/dl (12.5-16.5),
White cell count 3.6 x 109/L (4-11), Platelet
count 35 x 109/L (150-400). Select one of
following investigations which are most likely to
be abnormal?
1 )Antiphospholipid antibodies.
2 )Homocystine concentration
3 )Platelet function test
4 )Protein C concentration.
5 )Indium-labelled white cell scan.

A 30 year old of average height and weight presents with


polyuria and thirst. He has a blood glucose of 15 mmol/l.
There is no ketonuria and his older sister is diabetic
How should he be treated?
A. Start metformin
B. A fasting blood glucose should be sent before
treatment
C. Subcutaneous insulin should be started
D. Commence on gliclazide and reassessment with
BM monitoring at home
E. Dietary advice, review in a month with repeat
glucose without any treatment

Answer: d) commence on gliclazide and


reassessment with BM monitoring at
home. He is a type 1 diabetic but there are
no features of ketonuria or acidosis. He
may have some residual islet cell
function and hence sulphonylureas may
help to stimulate insulin production.

A 62-year-old man presented with difficult walking. He had a past


history of diabetes mellitus and cervical spondylosis, which had
required surgical decompression eight years previously. He drank
40 units of alcohol weekly. On examination there was fasciculation,
wasting and weakness in the left deltoid and biceps, with weakness
in the shoulder girdle muscles bilaterally. There was fasciculation in
the glutei and quadriceps bilaterally, weakness of hip flexion and
foot dorsiflexion, brisk reflexes in upper and lower limbs, and
extensor plantar responses. There was no sensory impairment.
What is the diagnosis?
1) alcoholic myopathy
2) diabetic amyotrophy
3) motor neurone disease
4) recurrent cervical cord compression
5) syringomyelia

The answer is 3
There are signs of lower (wasting, fasciculations)
and upper (brisk reflexes, extensor plantar
response) motor neuron involvement in the
presence of normal sensation. Motor neuron
disease is the commonest cause of such
presentation. Alcoholic myopathy and diabetic
amyotrophy do not share upper motor neuron
signs. Syringomyelia presents with sensory
symptoms and signs (spinothalamic). You expect
sensory involvement with cervical cord
compression.

78 years old man presents with an acute onset


of severe pain & swelling of the left wrist which
had developed since she had a chest infection
two weeks previously. On examination, he had a
temperature of 38 C,left wrist was red, swollen
& painful.
What is the appropriate investigation for this pt?
1)Erythrocyte sedimentation rate.
2 )Full bl. count .
3 )Joint aspiration.
4 )Serum urate concentration.
5 )X ray of joint.

A 30-year-old woman is evaluated in the


endocrinology clinic for increased urine output. She
weighs 60 kg and has a 24-hour urine output of
3500 ml. Her basal urine osmolality is 210
mOsm/kg. She undergoes a fluid deprivation test
and her urine osmolality after fluid deprivation (loss
of weight 3 kg) is 350 mOsm/kg. A subsequent
injection of subcutaneous DDAVP (desmopressin
acetate) did not result in a further significant rise of
urine osmolality after 2 hours (355 mOsm/kg).
Which of the following is the likely diagnosis?
A. Normal
B. Primary polydipsia
C. Osmotic diuresis
D. Pituitary diabetes insipidus
E. Nephrogenic diabetes insipidus

A 60 years old woman diagnosed with giant


cell arteritis was commenced on high dose
prednisolone therapy. What is the best
appropriate treatment for prevention of
steroid- induced osteoporosis?
1 )Bisphosphonate therapy
2 )Calcium vitamin D
3 )Hormone replacement therapy
4 )Raloxifene.
5 )Salmon Calcitonin.

A 19 year old girl presents at the antenatal clinic. She is


approximately six weeks pregnant and the pregnancy
was unplanned. She has a two year history of grand mal
epilepsy for which she takes carbamazepine. She has
had no fits for approximately six months. She wants to
continue with her pregnancy if it is safe to do so. She is
worried about her anticonvulsant therapy and the effects
on the baby and enquires how she should be managed?
1) Advise termination due to drug teratogenicity
2) Continue with carbamazepine
3) Stop carbamazepine until the second trimester
4) Switch therapy to phenytoin
5) Switch therapy to sodium valproate

The answer is 2
The patient and fetus are at far more risk from uncontrolled seizures
than from any potential teratogenic effect of the therapy. In
pregnancy total plasma concentrations of anticonvulsants fall and so
the dose may need to be increased. The potential teratogenic
effects (particularly neural tube defects) of carbamazepine do need
to be explained and in an effort to reduce this risk she should
receive folate supplements. Screening with AFP and second
trimester ultrasound are required. Vitamin K should be given to the
mother prior to delivery. There is no point in switching therapies as
this could precipitate seizures in an otherwise stable patient.
Similarly, both phenytoin and valproate are again associated with
teratogenic effects.

Which of the following investigations best


supports a diagnosis of new variant CJD:
1) CSF analysis
2) CT brain
3) EEG
4) EMG
5) MRI brain

The answer is 5
MRI brain typically shows bilateral posterior
thalamic high signal abnormalities in
patient with new variant CJD. EEG , CSF
analysis only shows non-specific changes.
EMG and CT brain are normal. Sporadic
CJD (and not new variant CJD) is
associated with specific EEG changes.

A 65 year old lady has ischaemic cardiomyopathy and


symptoms of breathlessness walking up one flight of
steps. Her breath sounds are clear. Chest X ray shows
cardiomegaly and clear lung fields. She is currently on
frusemide 40mg bd and perindopril 4 mg at night. What
medication should be added?
A. Spironolactone
B. Carvedilol
C. Digoxin
D. Amiodarone
E. Diltiazem

Answer: b) carvedilol. The two best options are


carvedilol and spironolactone. Both B blocker
trials (CIBIS II, Merit HF, Copernicus) and
spironolactone trials (RALES) have shown
symptomatic improvement and decreased
mortality. In this patient with little signs of fluid
overload, a beta blocker can be started first, and
then spironolactone added as well.

A 33 year old epileptic female presents with visual


problems. Examination reveals a constriction of
visual fields to confrontation. Which of the
following may be responsible for her visual
deterioration?
1) Vigabatrin
2) Lamotrigine
3) Gabapentin
4) Phenytoin
5) Sodium Valproate

The answer is 1
Vigabatrin is associated with constricted
visual fields and when detected therapy
should be stopped.

A 45-year-old attends clinic complaining of


tiredness. She is hypothyroid & takes
thyroxine 150 micrograms daily. Which of
following
useful test assessing
appropriateness of thyroid hormone
replacement in primary hypothyroidism?
1)Free T3 & T4 concentrations
2 )Skin biopsy
3 )Thyroid binding globulin
4 )Total T3 T4
5 )TSH

A 52 year old lady has palpitations. Her ECG


shows a broad complex tachycardia. Which of
these features suggests that the tachycardia is
more likely to be of ventricular origin?
A. QRS of 150 ms
B. Left bundle branch block and left axis
deviation
C. P wave for every QRS complex
D. History of atrial fibrillation
E. Heart rate of 150

Answer: a) QRS of 150ms. Features that


favour VT are : QRS of > 140ms,
dissociated p waves, history of ischaemic
heart disease, right bundle branch block
with left axis deviation, HR >170 beats per
minute.

A 59 year lady has a history of hypertension and


has recently been prescribed a new
antihypertensive agent. She now has fevers
and joint pains in the arms, shoulders, knees
and ankles. Which of the following drugs could
cause this?
A. Bendrofluazide
B. B. deltiazem
C. C. Methyldopa
D. D. Amlodipine
E. E. Lisinopril

Answer: c) methyldopa. The history would be


consistent with drug induced SLE. In druginduced lupus erythematosus, the features of
arthritis, systemic symptoms, and cardiac and
pulmonary (lung) symptoms may be present.
Other symptoms associated with SLE, such as
lupus nephritis and neurological disease, are
rare. The drugs which may cause this are
procainamide, isoniazid, chlorpromazine,
penicillamine, sulfasalazine, hydralazine,
methyldopa, and quinidine.

A 55 year old man has bitemporal hemianopia. He


also has elevated IGF-1 levels in the serum and an
enlarged pituitary seen on MRI. Which one of the
following is an associated feature?
A. Hypolipidemia
B. Hypoglycaemia
C. Rheumatoid arthritis
D. Myocarditis
E. Hypertension

Answer: e) hypertension. The diagnosis is


acromegaly. Carpal tunnel syndrome,
impaired glucose tolerance, carpal tunnel
syndrome and high cardiac output cardiac
failure are associated.

MRI showing a pituitary tumour in


acromegaly

An 18 year-old girl receives radioactive iodine


as treatment of thyrotoxicosis. Which of
following is likely long-term complication of
this treatment?
1)hypoparathyroidism.
2 )hypothyroidism.
3 )increased risk of developing cancer.
4 )recurrent laryngeal nerve damage.
5 )osteoporosis.

Which of following percentages


accurately reflects mortality associated
with modern management of diabetic
ketoacidosis?
1 )0.5%
2 )1%
3 )2-3%
4 )5-6%
5 )8-10%

A 30 year old lady has recurrent attacks of


dizziness and blackouts. A 72 hour fast
reveals periods where her plasma glucose is
2.5 mmol/l with elevated insulin and C
peptide levels. What is the next best
investigation?
A. Repeat 72 hour fast
B. B. Glucagon stimulation test
C. C. Glucose tolerance test
D. D. MRI of abdomen
E. E. Insulin antibodies

Answer: d) MRI of abdomen. The tests so


far suggest an insulinoma. Localisation of
the insulinoma can be done with MRI, CT,
superior mesenteric angiography or
pancreatic venous catheterisation.

Insulinoma

A 35 year old lady has a history of diarrhoea,


profuse sweating and irritability. Her parents
had similar symptoms and passed away in
their fourties after doing radical surgery .
Which one of the following is the most likely
diagnosis?
A. Phaeochromocytoma
B. Acromegaly
C. Diabetes
D. Ovarian carcinoma
E. Medullary thyroid carcinoma

Answer: e) medullary thyroid carcinoma.


Medullary thyroid carcinoma produces
peptides and neurohormones which lead
to symptoms of irritability, diarrhoea and
sweating. It can be inherited in association
with MEN type II.

A young woman has acne and is taking oral medication. She develops
polyarthritis and raised liver enzyme tests. Investigations show
AST 95
ALT 170
bilirubin 16
antinuclear antibodies strongly positive at 1/640, Which of the
following drugs is she most likely to have been prescribed?
1) erythromycin
2) isotretinoin
3) minocycline
4) oxytetracycline
5) trimethoprim

The answer is 3
Except trimethoprim all other drugs are used
in the treatment of acne. And all of these
can cause hepatotoxicity. Erythromycin
usually causes cholestasis. Minocycline
can cause drug induced SLE.

Which of the following features is


characteristic of early Alzheimer's
disease?
1) ataxic gait
2) impaired short term memory
3) myoclonic jerks
4) urinary incontinence
5) visual hallucinations

The answer is 2
Alzheimer's disease is characterised early in the
disease by short term memory loss. The other
features listed here would suggest an alternative
diagnosis such as normal pressure
hydrocephalus (gait ataxia and urinary
incontinence), Creutzfeld-Jacob disease
(myoclonic jerks) and delirium or vascular
dementia (visual hallucinations).

A 45-year-old man presents with an insidious onset of


binocular horizontal diplopia and left sided facial pain. On
examination ha has a left abducens nerve palsy and
numbness over the maxillary division of the left
trigeminal nerve. The most likely anatomical site of his
neurological lesion is:
1) Cavernous sinus
2) Petrous apex
3) Superior orbital fissure
4) Cerebellopontine angle
5) Midbrain

The answer is 2
In the pre-antibiotic era an abducens nerve palsy with
ipsilateral pain and numbness was due to petrous
osteitis (Gradenigo syndrome) but is now more likely the
result of a meningioma or nasopharyngeal carcinoma of
the petrous apex. The cavernous sinus syndrome
consists of variable involvement of: oculomotor,
trochlear, abducens, trigeminal (ophthalmic and maxillary
division) and oculo-sympathetic nerves. The superior
orbital fissure syndrome is similar to the cavernous sinus
syndrome except for the presence of proptosis.

2. A 75-year-old male was admitted with chest pain &


dyspnoea. His pain subsides, he is generally well,
although dyspnoea restricts his mobilisation. He is
unable manage the stairs, but can mobilise solely
around the ward. Whilst being monitored his
telemetry demonstrates short runs of non sustained
ventricular tacchycardia, associated lightheadedness.
What is the next appropriate investigation for this
patient?
1 )24hr tape
2 )Coronary angiography
3 )Echocardiography
4 )Electrophysiological studies
5 )Outpatient Cardiology referral

Comments: 3 )Echocardiography
patients with reduced LV function or
asymptomatic VT may benefit from
implantation of an implantable cardiac
defibrillator . You need to know the
patients cardiac function, so we feel that
an ECHO would be the next appropriate
study in this patient, followed by coronary
angiography & inpatient cardiology
referral.

A 55-year-old female receiving 10 mg of Methotrexate & 5mg


of folate* weekly presents with a sore right finger after cutting
herself in the garden. On examination, she has a swollen,
erythematosus right ring finger up to the proximal
interphalangeal joint & you diagnose a cellulitis. You give her a
prescription of erythromycin as she is allergic to penicillins.
She has being receiving the Methotrexate over just one year
with no problems in all routine bl monitoring which are
normal.
Whilst monitoring the response of the infection to treatment,
what
appropriate strategy regarding her Methotrexate
therapy?
1 )Continue Methotrexate unchanged & increase folate
supplements to 10mg daily.
2 )Continue Methotrexate & folate unchanged.
3 )Reduce dose of Methotrexate to 5mg weekly
4 )Stop Methorexate until the infection is resolved.
5 )Stop Methotrexate only if full bl. count reveals a neutropaenia.

Comments:
true 4. In the circumstances of infection, one should
consider temporarily stopping methotrexate as it is an
immunosuppressant. Any infection should be treated as
usual to response to treatment monitored. Once the
infection is successfully treated methotrexate can be
reinstated. However, if the patient has recurrent serious
infections while taking methotrexate, its continued long
term use should be discussed with the patient's
rheumatologist. *some local variations may exist
regarding dose & frequency of folate therapy.

14.A 26-year-old man with a history of alcohol &


drug abuse was admitted with a 14 day history
of fever, cough & fatigue. He was emaciated.
His temperature was 39.4C. Cervical & axillary
lymphadenopathy were present. Chest X-ray
showed bilateral areas of pulmonary
shadowing. Which of the following is the likely
diagnosis?
1 )alcoholic cardiomyopathy
2 )pneumococcal pneumonia
3 )pneumocystis pneumonia
4 )pulmonary tuberculosis
5 )tricuspid endocarditis

A 45 year old woman with breathlessness


presents for further investigation. She has a
known history of rheumatoid arthritis and is
on methotrexate and folate. She has a CXR
and lung function tests. Which one of the
following is a recognized respiratory
manifestation of rheumatoid arthritis?
A. Mesothelioma
B. B. Asthma
C. C. Bronchiolitis obliterans
D. D. Pulmonary eosinophilia
E. E. Pulmonary embolus

Answer: c) bronchiolitis obliterans. In


rheumatoid arthritis, exudative pleural
effusions, fibrotic lung disease, Caplans
syndrome (pneumoconiosis, pulmonary
nodules), and obstructive lung disease in the
form of bronchiolitis obliterans (obstruction of
bronchiolar lumen with inflammatory exudate)
may occur. Pulmonary eosinophilia may be
found in Churg Strauss or Wegeners vasculitis.

A 55 year old female has been on long-term steroids for


chronic obstructive pulmonary disease. She complains of
pain in her right groin radiating down the anteromedial
thigh. On examination of the hip, there is decreased
range of movement especially flexion, abduction and
internal rotation. What is the likely diagnosis?
A. Osteoarthritis
B. Rheumatoid arthritis
C. Metastatic hip lesion
D. Avascular necrosis of the femoral head
E. Hairline fracture

Answer: d) avascular necrosis of the femoral head.


In a patient on long term steroids presenting with groin
pains radiating to the thigh associated with an antalgic
gait and decreased range of movement of the hip, the
most likely diagnosis is avascular necrosis of the
femoral head.
In this condition, MRI is the most sensitive and specific
technique and is useful for early diagnosis before
collapse of bone occurs. CT scan and x-ray are useful to
rule out advanced disease if duration is not clear. Bone
scanning is more sensitive than x-ray but is non-specific.

MRI showing avascular necrosis of the femoral head

A patient who is 8 months pregnant has a thyroid


function test because she has a smooth
palpable goitre. Her Free T4 is 20 pmol/l and
TSH is 5 mU/l. Which of these diagnoses is most
likely?
A. Medullary thyroid carcinoma
B. Multinodular goitre
C. Grave's disease
D. Thyroid adenoma
E. Colloid goitre

Answer: e) colloid goitre. A colloid goitre is most


likely. Presentation is with mildly elevated TSH
(0.3-4.0 mU/l) and normal free T4. Mild iodine
deficiency or a pregnancy can cause the goitre
to become more noticeable.

Colloid Goitre -increased size of follicles and


flattening of follicular epithelial cells

A 55 year old patient with type 2 diabetes is


reviewed in the diabetic clinic. Which of
the following is a feature of diabetic
neuropathy to watch out for?
A. Cervical myelopathy
B. B. Brisk reflexes
C. C. Muscle hypertrophy
D. D. Loss of vibration sense
E. E. Myotonia

Answer: d) loss of vibration sense. Autonomic


neuropathy to the gut, bladder and sexual
organs (impotence) can occur. A 3rd nerve
mononeuropathy can occur. Motor neuropathy
can cause muscle wasting, and sensory
neuropathy causes vibration sensory loss. With
myotonia, prolonged contraction of muscle fibres
associated with muscle dystrophy is due to
genetic causes of muscle protein abnormality
and is not neurologically dependent.

A 40 year old lady has a pulmonary systolic


murmur which is louder on inspiration. She
also has a right ventricular heave. The second
heart sound is wide and fixed with splitting.
What is the likely diagnosis?
A. Mitral stenosis
B. Aortic stenosis
C. Atrial septal defect
D. Dilated cardiomyopathy
E. Hypertrophic cardiomyopathy

Answer: c) atrial septal defect.


In Atrial Septal Defect (ASD), a Primum defect
causes RBBB and LAD, whilst Secundum
causes RBBB and RAD on the ECG. A systolic
murmur is heard in the pulmonary area because
of increased pulmonary valve flow due to
pulmonary hypertension. Similarly, a left
parasternal heave is present due to RVH.
ECG showing RBBB and LAD

A 75 year old man with known moderate to severe


aortic stenosis presents to hospital with
breathlessness. Which of the following
features would indicate poor prognosis without
urgent treatment?
A.Early diastolic murmur
B. Displaced apex beat
C. Pulmonary oedema
D. Aortic valve calcification seen on X ray
E. Breathlessness

Answer: c) pulmonary oedema. Pulmonary


oedema and angina can occur with aortic
stenosis, these are symptoms which
indicate urgent assessment should be
made with a view towards surgery.

A 45 year old man has polyuria and abdominal


pains. His calcium is 2.9 mmol/l, phosphate 0.8
mmol/l and PTH 12 pmol/l. Which one of the
following would suggest that he has primary
hyperparathyroidism?
A. Serum alkaline phosphatase is increased
B. Normal skull
C. Arthritis
D. Low urinary phosphate excretion
E. Low urinary cAMP

Answer: a) serum alkaline phosphatase is increased.


Serum alkaline phosphatase is usually increased in
primary hyperparathyroidism.
Primary hyperparathyroidism does not respond to
steroids.
Salt and pepper changes on the skull can be a hallmark.
Gout, rather than arthritis, is worsened by
hyperparathyroidism.
High urinary phosphate excretion leads to serum
phosphate and a high cAMP in the urine suggests active
excretion.

A 40 year old lady has increasing hirsutism. She


is embarassed about having to shave her chin
and also her chest. Her voice is becoming
deeper. What is the most likely diagnosis?
A. Drug induced hirsutism .
B. Polycystic ovarian syndrome.
C. Adrenal tumour.
D. Congenital adrenal hyperplasia.
E. Ovarian carcinoma .

Answer: c) adrenal tumour. Rapid


development of hirsutism is usually
caused by an adrenal tumour. There are
high testosterone or DHEA levels in the
plasma.

MRI showing an adrenal tumour

A 60 year old man has worsening joint stiffness,


hip and back pains. He has spinal and hip X rays
for assessment. Which one of the following is a
recognised X ray change of osteoarthritis?
A. Osteitis fibrosaB.
B.Lytic lesions
C. Fibrosis
D. Subchondral cysts
E. Hairline fracture

Answer: d) subchondral cysts.


Osteophytes, loss of joint space,
subchondral sclerosis and subchondral
cysts are common in osteoarthritis.

A 55 year old lady has abdominal pains. She


has polyuria and polydipsia. Her free T4 is
16 pmol/l and TSH is 3.0 mU/l. Which of the
following is most likely?
A. Subclinical hypothyroidism
B. Renal tubular acidosis
C. MEN 1
D. Diabetes
E. Primary hyperparathyroidism

Answer: e) primary hyperparathyroidism.


Hypercalcaemia can cause abdominal
pain, polyuria and polydipsia. There are no
features suggestive of pituitary tumour,
phaeochromocytoma or medullary thyroid
involvement, hence MEN is unlikely.

A 10 year old child with joint pains is suspected


of having juvenile chronic arthritis. A full
rheumatological history is taken. Which of the
following is a recognised presenting feature
of juvenile chronic arthritis?
A. Low grade fever
B. Transient purpuric rash
C. Haematuria
D. Involvement of one or two large joints only
E. Ulcerative colitis

Answer: d) involvement of one or two large joints


only. Juvenile chronic arthritis is rheumatoid
factor negative. Commonest type is Stills
disease. Peaks of disease are about 5 years and
15 years of age. A high swinging fever is typical.
An erythematous rash can occur, but it is non
purpuric. Haematuria is not typical. Certain
forms can involve only one or two large joints
only. Different classifications are systemic,
pauciarticular and polyarticular. There is no
relation with ulcerative colitis.

La boratory evaluation of a 19-year-old male who is being worked up for


polyuria and polydipsia yields the following results:
S. Na+ 144, K+ 4.0,
Cl 107,
HCO3 25
BUN: 6.4 mmol/L.
Bl. gl: 5.7 mmol/L , Urine electrolytes (mmol/L): Na+ 28, K+ 32 Urine
osmolality: 195 mosmol/kg water After 12 h of fluid deprivation, body weight
has fallen by 5%. Laboratory testing now reveals the following:
Serum electrolytes (meq/L): Na+ 150, K+ 4.1, Cl 109, HCO3 25 BUN: 7.1
mmol/L (20 mg/dL) (98 mg/dL) Urine electrolyte : Na+ 24, K+ 35 Urine
osmolality: 200 mosmol/kg water One hour after the subcutaneous
administration of 5 units of arginine vasopressin urine values are as follows:
Urine electrolytes (meq/L): Na+ 30, K+ 30 Urine osmolality: 199 mosm
The likely diagnosis is
A. nephrogenic diabetes insipidus
B. osmotic diuresis
C. salt-losing nephropathy
D. psychogenic polydipsia
E. none of the above

Slide no 6

This female patient sufferred convuslions in childhood.


She experienced several attacks of hematuria recenly.
Proteinuria was less thn 1gm/24hrs and her RFTs
were normal.
1. What is her condition.
2. What is the most likely lesion of her CT scan

A 50 year man presents with lightheadness. He has


frequent nonsustained ventricular tachycardia on the
ECG and cardiac monitor. His bloods show a Hb 13.0
g/dl, WCC 7 x 10^9/l, platelets 230 x 10^9/l, urea
11mol/l, creatinine 80mol/l, sodium 134 mmol/l,
potassium 3.2 mmol/l, serum magnesium of 0.6 mmol/l
(0.75). Which one of following is likely to be responsible
for his arrhythmias?
A. Poor diet
B. Alcoholism
C. Frusemide
D. Diarrhoea
E. Hyperphosphataemia

Answer: c) frusemide. The likely cause of


the arrhythmias is hypomagnesaemia and
hypokalaemia, which is most commonly
associated with diuretic use.

A 38 year old lady presents with myalgia and


lethargy. Her blood tests show a positive ANA
with a titre of 1:1024 and rheumatoid factor is
negative. The CK is raised at 360 U/l.
Extranuclear antigen tests show a negative
Ro and negative La, negative Scl70 and
positive ribonuclear protein antibody at 160
units. What is the likely diagnosis?
A.Polymyalgia rheumatica.
B. Polymyositis.
C. Scleroderma.
D. Systemic lupus erythematosus .
E. Mixed connective tissue disease.

Answer: e) mixed connective tissue


disease. A positive ANA (speckled
pattern), raised CK and positive anti RNP
antibody suggests mixed connective
tissue disease.

A 29-year-old banker is said to have gone camping


three weeks ago. He reports having developed a skin
rash around the groin area. It began as a papule but
went on to become a macule with a bright outer
border and a clear centre. It was warm but not
painful. The macule is disappearing, but he has since
developed severe headache, mild neck stiffness,
fever chills, pains which were once in the knees but
are now in the ankles and elbows. He also has
malaise and fatigue.
What is the most probable diagnosis?
Infective endocarditis
Lyme disease
Meningococcal meningitis
Q fever
Rheumatic fever

A 40 year old man has genital ulceration and


uveitis. his GP suspects Behcet's syndrome
and is referred to the rheumatologist. Which
one of the following is a feature of Behcets
syndrome?
A. Facial asymmetry
B. Receding hair line
C. Hirsutism
D. Malignant melanoma
E. Arterial thrombosis

Answer: e) arterial thrombosis. Behcet's


syndrome is an inflammatory disorder
(associated with certain HLA B and DR types)
causing mouth ulceration, arthritis, eyes (anterior
uveitis, retinal vein occlusion), vasculitis
(thrombophlebitis) and thrombosis. CNS
vasculitis involvement may lead to TIA,
meningoencephalitis, parkinsons and dementia.

A 42-year-old male a 15 year history of type 1


diabetes presents a two month history of
deteriorating pain & stiffness of the right shoulder.
On examination he has painful limitation of internal
rotation but can abduct right arm only 90 degrees.
Flexion is relatively unimpaired. There some
weakness of movement of shoulder slight wasting
of shoulder muscles. He has some reduced vibration
sensation in both hands.
Which of following is likely diagnosis?
1 )Adhesive capsulitis
2 )Brachial plexopathy
3 )Calcium pyrophosphate arthropathy
4 )Diabetic arthropathy
5 )Rheumatoid arthritis

A 70 year old lady presents with bilateral shoulder


and neck pains which have been present for
several years. She also feels constantly
lethargic. Her ESR is 105 mm/hour. What is
the likely diagnosis?
A. Fibromyalgia
B. Supraspinatus tendinitis
C. Polymyalgia rheumatica
D. Temporal arteritis
E. Cushing's syndrome

Answer: c) Polymyalgia rheumatica. With


a raised ESR, polymyalgia rheumatica is
far more likely than fibromyalgia. Temporal
arteritis should always be considered
especially if a headache is present.

A 50-year-old male is admitted to the hospital with


pneumonia. He does well after the administration of
antibiotics,but his sodium is noted to rise from 140 to 154
meq/L over 2 days. He reports thirst and has had a urine
output of approximately 5 L per day. Which of the
following is the most appropriate next step to evaluate
the patients disorder?
A. Measurement of serum osmolality
B. Measurement of serum vasopressin level
C. 24-h measurement of urinary sodium
D. Trial of arginine vasopressin
E. Trial of free water restriction

A 55 year old patient presented with breathlessness and ankle oedema. The
blood pressure is 135/80 mmHg. On examination, her JVP rises with
inspiration. She has a soft systolic murmur and a third heart sound.
Blood tests reveal
Hb 10.5 g/dl
WCC 7.5 x 10^9/l
Platelets 150x 10^9/l
sodium 136 mmol/l
potassium 3.5 mmol/l
creatinine 140 mol/l
urea 6 mol/l
ECG shows poor R wave progression. An echocardiogram shows no
pericardial effusion, the ventricles are stiff and systolic function is mildly
impaired.
Which of the following is the likely diagnosis?
A.Restrictive cardiomyopathy
B. Dilated cardiomyopathy
C. Constrictive pericarditis
D. Ischaemic cardiomyopathy
E. Pulmonary embolus

Answer: a) restrictive cardiomyopathy. In this


scenario, there symptoms can be caused by any
form of cardiomyopathy. The rise in JVP with
inspiration suggests either constrictive or restrictive
cardiomyopathy. Echocardiography showing no
pericardial effusion and stiffness suggests restrictive
rather than constrictive cardiomyopathy. The
transmitral dopplers on the echo may show E/A
wave reversal and high velocities which may
suggest restrictive picture. This may be due to
infiltration due to haemochromatosis,
endomyocardial fibrosis, sarcoidosis, myeloma,
lymphoma or connective tissue disease.

A 35 year old female presents with


sweatiness, tremors and palpitations.
Examination reveals a exopthalmos and a
goitre. Her GP requests TFTs which show
a TSH of 0.01mU/l, FT4 35 pmol/l, FT3 3.1
nmol/l. She has positive antithyroid
antibodies. What is the likely diagnosis?
A. Hashimoto's thyroiditis
B. Grave's disease
C. Iodine deficiency
D. Post radioiodine treatment
E. Papillary thyroid carcinoma

Answer: b) Grave's disease.


Graves disease is the diagnosis - thyroid
autoantibodies are increased. Almost 80% of
patients have exopthalmos. Medical treatment
such as carbimazole or radioiodine treatment
are recommended rather than surgery. There will
be increased uptake on the thyroid radioisotope
scan.

Exopthalmos in Grave's disease

A 68-year-old female is referred to outpatients with a


three-month history of dyspnoea and significant
peripheral oedema. Clinical findings are solely of
the oedema. Echocardiography demonstrates
thickened myocardium with impaired relaxation and
a bright speckled appearance.
What is the likely diagnosis?
A : Hypertrophic cardiomyopathy.
B : Cor pulmonale.
C : Sarcoidosis.
D : Amyloidosis.
E : Secondary neoplastic myocardial deposition.

A 44-year-old woman presents to her GP for review.


She has attended the GP on a number of
occasions during the past year for sinusitis and is
now concerned that the bridge of her nose has
collapsed and that she may require cosmetic
surgery. Chest X-ray is abnormal and reveals
multiple nodules. Routine blood testing reveals a
creatinine of 205 mol/l. Renal biopsy reveals a
necrotising microvascular glomerulonephritis.
Which of the following is the best initial
treatment for this condition?
1. Corticosteroids
2. Cyclophosphamide
3. Corticosteroids and cyclophosphamide.
4. Azathioprine
5. Methotrexate

True 3. The clinical history is highly suggestive


of Wegeners Granulomatosis. Often patients
present with severe rhinorrhoea, complicated
by nasal ulceration and later cough with
haemoptysis. Chest X-ray reveals nodular
changes +/ pneumonic features. Renal
involvement is characterised by
microvascular glomerulonephritis. Treatment
is with cyclophosphamide in combination
with corticosteroids in patients with severe
disease.

A 45-year-old woman presented with a 1-year history of


intermittent swelling and pain in the small joints of her
hands. Maximal stiffness occurs on waking and eases
after an hour. On examination rheumatoid nodules on
the elbows and symmetrical soft tissue swelling over the
PIP and MCP joints were noted. Effusions of both wrists
were also noted. She is rheumatoid factor positive.
Radiological examination of the wrist and hands showed
erosions and bony decalcification. Her GP has
commenced her on NSAIDs and she been referred to a
rheumatologist for consideration of DMARD. She has a
past history of TB.
Which of the following is best avoided if possible?
1. Methotrexate
2. Chloroquine
3. Sulfasalazine
4. Infliximab
5. leflunomide

True 4. Anti-TNF-a is an effective


treatment for rheumatoid arthritis (RA).
It has been shown to give a sustained
clinical response over 2 years.
However, it can reactivate old TB and
should be used with caution in these
patients.

A patient with a history of Sjgrens syndrome has


the following laboratory findings: plasma sodium 139
meq/L, chloride 112 meq/L, bicarbonate 15 meq/L, and
potassium 3.0 meq/L; urine studies show a pH of 6.0, sodium
of 15 meq/L, potassium of 10 meq/L, and chloride
of 12 meq/L. The most likely diagnosis is
A. type I renal tubular acidosis (RTA)
B. type II RTA
C. type III RTA
D. type IV RTA
E. chronic diarrhea

72-year-old male presents a 2 month history of


weight loss & weakness. Examination reveals a
BMI of 24.5 kg/m2 a pressure of 186/100 mmHg.
Examination of lower limbs reveals a bilateral
weakness of knee extension. He is unable to rise
from squatting position. There absence of knee
reflex but ankle reflexes are preserved both
plantars are flexor. There are no abnormalities on
sensory examination.
Which of following tests may be diagnostic?
1 )Vitamin B12 concentration
2 )Thyroid function test
3 )Oral glucose tolerance test
4 )Urine free cortisol concentration
5 )Vitamin D concentration

Which of the following would be correct in


keeping with a diagnosis of ploymyalgia
rheumatica?
1 )raised creatinine kinase
2 )increased alkaline phosphatase
3 )sudden loss of vision in one eye
4 )shoulder & pelvic girdle pain in 35-yearold man
5 )erythema nodosum

Correct answer 2

A 45-year-old woman admitted a spiking temperature


sweats. She unwell last 3 weeks flitting arthralgia
lethargy. There a rash over her trunk which prevalent
in mornings. Blood cultures are sterile. Her recent
transthoracic echocardiogram normal. ESR 56mm/hour.
Her ferritin elevated at 6000(g/l. Autoimmune screen
negative.
1 )bacterial endocarditis.
2 )systemic lupus erythematosus .
3 )rheumatoid arthritis
4 )adult onset Stills disease .
5 )meningitis .

A 52-year-old schoolteacher attends for weight loss & sweats.


Subsequent investigations show:
Free T4 of 40 pmol/L (9-23)
Free T3 ofs 9.8 nmol/L (3.5-6)
TSH of 7.0 mU/L (0.5-5)
She is clinically thyrotoxic a diffuse goitre & commenced on
Propranolol & Carbimazole.

What is appropriate test this patient?


1 )FNA of thyroid gland
]2 )MRI scan pituitary gland
3 )Radio-isotope uptake scan of thyroid gland
4 )Repeat TFT
5 )Thyroid auto antibodies

A 75-year-old man presents with 12 months history


of cognitive impairment, parkinsonism, intermittent
confusion and generalised myoclonus. He was
started on 62.5 tds of sinemet. In the following 2
months he was started experiencing visual
hallucinations.
The most likely diagnosis is:
1) Idiopathic Parkinson's disease
2) Alzheimer's disease
3) Diffuse Lewy body disease
4) Multiple system atrophy
5) Progressive supranuclear palsy

The answer is 3
Diffuse lewy body disease presents with cognitive
impairment, visual hallucinations, intermittent
confusion, parkinsonism, myoclonus and marked
sensitivity to neuroleptic treatment. Visual
hallucinations in parkinson's disease treated with
L-dopa usually appear late (>2 years after
initiation of treatment). Visual hallucinations are
not features of multiple system atrophy or
progressive supranuclear palsy.

A female patient aged 30 has a 5 years history of difficulty


getting upstairs and out of a low chair and mild upper
limb weakness but no pain. There is no family history.
She presented with severe type 2 respiratory failure.
EMG showed evidence of myopathy. The most likely
diagnosis is:
1) Polymyositis
2) Inclusion body myositis
3) Acid Maltase Deficiency
4) Miller-Fisher Syndrome
5) Lambert-Eaton Myasthenic Syndrome

The answer is 3
Acid maltase deficiency typically presents with insidious
onset of proximal myopathy and early respiratory muscle
weakness. Respiratory failure in inflammatory
myopathies (polymyositis, dermatomyositis, inclusion
body myositis) and limb girdle muscular dystrophy is
rare. Muscle biopsy shows vacuolation in muscle fibres.
Miller-Fisher Syndrome, a variant of GBS, is
characterised by ophthalmoplegia, ataxia and areflexia.
Lambert-Eaton Myasthenic Syndrome, often a
paraneoplastic phenomenon, is associated with
hyporeflexia which returns after exercise, autonomic
symptoms and fatiguability.

Which statement is true regarding Gabapentin?


1) is a potent hepatic enzyme inducer
2) side effects typically include visual field defects
with long-term use
3) therapy is best monitored through measuring
plasma concentrations
4) is of particular value as monotherapy in
absence attacks (petit mal)
5) requires dose adjustment in renal disease

The answer is 5
Gabapentin does not induce cytochrome P450
unlike other anticonvulsants such as phenytoin
and phenobarbitone. Vigabatrin may cause
visual field defects, which may be irreversible.
Rarely have visual disturbances been
associated with gabapentin. No use in Petit Mal
and is used for add-on therapy in partial or
generalised seizures.

Which of the following clinical manifestations


suggests Guillain Barr Syndrome?
1) Weakness beginning in the arms
2) Asymmetrical involvement of distal muscles
3) Bulbar involvement in about 50% of cases
4) Brisk tendon reflexes
5) Normal CSF protein

The answer is 3
GB is a post-infectious polyneuropathy causing demyelination in mainly motor
but also sensory nerves. It usually follows a non-specific viral infection.
Campylobacter and mycoplasma are recognised causes. Weakness begins
in the legs and progressively ascends to involve the trunk, upper limbs and
finally the bulbar muscles (Landry's ascending paralysis). Asymmetry is
present in only 9% of patients, with symmetrical involvement being typical.
Usually there is painless progression over days or weeks, but in cases of
abrupt onset, there may be tenderness or muscle pain. Bulbar involvement
occurs in 50%, with a risk of aspiration and respiratory insufficiency can be
problematic. In the Miller Fisher Syndrome there is external
ophthalmoplegia, ataxia and areflexia. In 20% of cases there is urinary
incontinence of retention. Clinical symptoms usually improve within 2-3
weeks, though a chronic relapsing form is recognised. CSF protein is
elevated to more than twice the upper limit of normal, with normal glucose
and no pleocytosis. Bacterial cultures are negative and viral cultures rarely
isolate anything. The dissociation between a high CSF protein and a lack of
cellular response in a person with an acute or subacute polyneuropathy is
diagnostic of Guillain Barr Syndrome

A 72-year-old lady has 4 months of memory loss,


urinary incontinence and falls. On examination
she has mild memory loss and a broad-based,
slow gait. Muscle tone is normal and both
plantar reflexes are downgoing. What is the
likely diagnosis?
1) Alzheimer's disease
2) Frontal lobe dementia
3) Mulit-infarct dementia
4) Normal-pressure hydrocephalus
5) Parkinson's disease

The answer is 4
Normal pressure hydrocephalus characterized by
abnormal gait, urinary incontinence, and
dementia. It is an important clinical diagnosis,
because it is a potentially reversible cause of
dementia. It is important to distinguish it from
Parkinson's Disease. The onset of gait
disturbance and urinary symptoms is unusual so
early in dementia. Frontal lobe dementia is
characterised by loss of 'executive' functions and
multi-infarct state usually has a step-wise
history.

A patient has, on examination, weakness in


plantar flexion and foot inversion on the left. He
also is unable to tiptoe on the same foot. Ankle
jerk is absent. Which of the following nerve
lesion is most likely?
A. Common peroneal nerve
B. L4 nerve root
C. Tibial nerve
D. Sciatic nerve
E. Femoral nerve

Answer: c) tibial nerve. Tibial nerve


supplies the gastrocnemius muscle and
leads to the above findings. The common
peroneal nerve causes weakness of
eversion and dorsiflexion.

A 25 year old secretary has had several episodes of brief


jerking of the right arm over the past few weeks. There is
no loss of consciousness. A CT scan of the head is
unremarkable. Which is the best medication to
commence?
A. Carbamazepine
B. Phenytoin
C. Lorazepam
D. Diazepam
E. Levodopa

Answer: a) carbamazepine. Brief episodes


of jerking suggests simple partial seizures.
Carbamazepine is first line therapy for
this.

A 40 year old patient has a transthoracic


echocardiogram as a follow up. He has a diagnosis
of hypertrophic obstructive cardiomyopathy. Clinically
he has a systolic murmur heard loudest in the right
upper sternal edge. His ECG shows grossly large
QRS complexes with LVH strain pattern. Which of
the following suggests highest risk for sudden death?
A left ventricular outflow tract gradient of 20
mmHg
B. Tricuspid regurgitation
C. Systolic anterior motion of mitral valve
D. ECG showing ventricular ectopics
E. Interventricular septal thickness of 4 cm

Answer: e) interventricular septal


thickness of 4 cm. The LV outflow tract
gradient is not significantly high in this
patient, however, this may vary with
exercise. The large interventricular septal
thickness of 4 cm (normal <1.3 cm)
suggests very hypertrophic myocardium
and high risk of outflow tract obstruction
with exertion.

A 18 year old man is referred for an


echocardiogram for further investigation of
a systolic murmur. This reveals a bicuspid
aortic valve. Which of the following is an
association?
A. Mitral stenosis
B. Mitral valve prolapse
C. Marfan's syndrome
D. Down's syndrome
E. Coarctation of aorta

Answer: e) coarctation of aorta. Bicuspic


aortic valve can be congenital. 5% of
cases demonstrate significant association
with coarctation of the aorta.

Coarctation of Aorta

A 50 year old woman with rheumatoid arthritis


has hand and spine X rays due to worsening
joint and back pains. Which of the following is
a recognised X ray change of rheumatoid
arthritis?
A. Juxta-articular osteosclerosis
B. Sacroilitis
C. Tendon swelling
D. Marginal erosions
E. Calcification of entheses

Answer: d) marginal erosions. Juxta-articular


osteoporosis or osteosclerosis occurs in OA.
Sacroilitis is seen in seronegative arthritis.
Marginal erosions are seen at the articular
cartilage and attachment of synovium. There is
associated loss of joint space. Calcification of
entheses also occurs in seronegative arthritis.

Marginal Erosions of the first MTP joint

A 35 year old lady has positive ANA,


and has a butterfly shaped rash on
her face. Her physician makes a
diagnosis of SLE. She has flare ups
of joint swellings and pains requiring
several months treatment with
prednisolone. 1 year later she
presents with hip pain limiting her
mobility. Which one of the following is
the likely cause?
A. Rheumatoid arthritis
B. Septic arthritis
C. Juvenile chronic arthritis
D. Avascular necrosis
E. Perthe's disease

Answer: d) avascular necrosis. 15% of


patients with SLE develop avascular
necrosis of the bone. Nephritis, vasculitis
and long term steroid use predispose to
avascular necrosis. Collapse of the
femoral head due to avascular necrosis

A 65 year old man has impaired hearing in his


left ear, an enlarged skull vault, bowing of the
left tibia which is warm to touch. In this
patient, one of the complications of
immobilisation would be:
A. Osteoarthritis
B. Hypercalcaemia
C. Peripheral vascular disease
D. Venous varicosities
E. Disseminated intravascular coagulation

Answer: b) hypercalcaemia. The patient has the


clinical features of Pagets disease of bone. In
this condition immobilisation is likely to cause
hypercalcaemia. Other complications are gout,
high output cardiac failure and coincidental
hyperparathyroidism.

Bowed tibia in Paget's disease

A 60 year old lady has an autosomal dominant


condition. She small red lesions on her face and
mouth. She has been on ferrous sulphate tablets
for chronic iron deficiency anaemia. In view of the
likely diagnosis, which feature may be
associated?
A. Oesophageal dysmotility
B. Pulmonary AV malformation
C. Pulmonary fibrosis
D. Hiatus hernia
E. Aortic aneurysm

Answer: b) pulmonary AV malformation. She has


hereditary haemorrhagic telangiectasia (OslerRendu-Weber syndrome). Patients have a
chronic problem with recurrent GI bleeding and
require multiple transfusions and iron
supplementation. 10% of patients have
pulmonary AV malformation. Oesophageal
dysmotility is seen in scleroderma.

Telangiectatic lesions in HHT

A cardiothoracic surgeon is planning for surgery


in a 60 year old man with coronary artery
disease. In coronary artery bypass grafting,
which of the following vessels should be
considered for grafting to the Left anterior
descending artery?
A. Saphenous vein graft
B. Left internal mammary
C. Right internal mammary
D. Radial artery
E. Brachial artery

Answer: b) left internal mammary. The left


internal mammary artery supplies the anterior
chest wall. It has been shown to be superior to
saphenous vein grafts (from aorta to LAD) in
staying patent and hence is now the choice
artery (LIMA to LAD) graft. Although circumflex
and right coronary arteries are usually grafted
with veins, RIMA arteries are sometimes used to
graft the RCA.

A 45 year old man has developed pains and swelling over


his wrist joints bilaterally over 3 months. He has limited
wrist movements and clubbing. X rays show periosteal
reaction at the ends of the radius and ulnar bones
suggestive of periostitis. Which medical condition
predisposes to this?
Behcet's disease
B. Crohn's disease
C. Amyloidosis
D. Diabetes
E. Hyperthyroidism

Answer: b) Crohn's disease. Hypertrophic


osteoarthropathy (or hypertrophic pulmonary
osteoarthropathy when there is mesothelioma or
bronchogenic carcinoma associated) is
associated with conditions such as liver
cirrhosis, ulcerative colitis, whipple's disease
and crohn's disease.

Hypertrophic osteoarthropathy

A 30-year-old woman complains of severe headache, mainly in the occipital


region, as well as irritability, low mood and lack of energy. Her husband
informs you that he has noted her to be withdrawn and short tempered. She
also gives a history of a recent increase in hair loss, and joint pains,
predominantly in the wrist and knees. She has had amenorrhoea for the last
3 months. She works as a secretary, and is now finding it extremely difficult
to cope with her work and household activities. She and her husband are
requesting sick leave. On examination, she looks unwell. Her temperature is
38oC, she is pale and there is an erythematous rash on her face. Joint
examination is remarkable for tenderness in the wrists and knees, but no
swelling. Cardiovascular, Respiratory and abdominal examination are
unrevealing. Neurological examination does not show focal neurologic signs
or neck stiffness. Laboratory results were returned as follows: Hb 9.7
g/dl WBC3.8 x 10 9/l, N2.1 x 10 9/l,L0.8 x 10 9/l, PLT88 x 10 9/l, ESR 88
mm/1st hr, CRP8 mg/l, AST 44 IU/l, ALK-P156 IU/l, ALT 50 IU/l, ANA
positive, DNA and ENA awaited, Urine dipstick protein+
What is the most likely clinical problem that may explain the above?
1. Neuropsychiatric lupus
2. Viral meningitis
3. Lupus glomerulonephritis
4. Antiphospholipid antibody syndrome
5. Cranial angitis

True 1. The above clinical findings and investigation results are


suggestive of systemic lupus erythematosus (SLE). However,
the symptoms of headache and mood changes point towards
the possibility of neuropsychiatric lupus. Features of
neurological disease in lupus range from the common,
relatively harmless migraine headache, to major psychotic
episodes and grand mal seizures, recognised in some lupus
patients.
Lupus glomerulonephritis cannot be completely excluded but
active urinary sediment with proteinuria of ++ on dipstick will
be more suggestive.
Antiphospholipid antibody syndrome (APS) is unlikely even
though the patient has thrombocytopenia; lymphocytosis
rather than lymphopenia would be more consistent with APS.

A 65 year old man is assessed on the ward for weakness


in his legs. He is an ex smoker and drinks 15 units of
alcohol in a week. His wife mentions that he is confused.
On examination, his MMSE score is 20/30. He has an
ataxic gait. There is bilateral pyramidal weakness and
coordination is impaired. Routine blood tests are normal.
An MRI scan of the head shows diffuse white matter
changes, more in the cerebellar region than the
cerebrum. Which of these tests would help most in
confirming the diagnosis?
A. CSF for oligoclonal bands
B. CSF for anti Hu and anti Yo antibodies
C. CSF for TB culture
D. EEG E. EMG

Answer: b) CSF for Anti Hu and anti Yo


antibodies. Anti Hu and anti Yo antibodies
would help confirm a diagnosis of
paraneoplastic syndrome. Multiple
sclerosis is unlikely in view of late
presentation and is not commonly
associated with dementia.

A 36 year old female gave a recent history of


sensory impairment and imbalance in left
half of the body. She also complained of
tingling .She had recurrent episodes of
ataxia in last year each episode resolved
spontaneously. What is the likely diagnosis
of this episode?
A. Transient ischaemic attack
B. Intracranial space occupying lesion
C. Multiple sclerosis
D. Cerebellar haemorrhage

Answer: C) Multiple sclerosis.


History of patchy sensory loss, and ataxia
which improves on occasion is suggestive
of an inflammatory disorder, in this case
most likely multiple sclerosis.

Which of the following diseases is


associated with an increased frequency
of HLA -DR4 on the cell membranes?
A. Ankylosing spondylitis
B. B. Rheumatoid arthritis
C. C. Porphyria
D. D. Congenital adrenal hyperplasia
E. E. Narcolepsy

Answer: b) rheumatoid arthritis. Lack of


HLA DR2 is associated with narcolepsy,
and increased HLA DR4 is associated with
rheumatoid arthritis.

11.Anti-neutrophilic cytoplasmic autoantibodies:


A positive only in Wegeners syndrome associated with
renal disease .
B cause neutropenia in SLE .
C present in inflammatory bowel disease .
D is invariably associated with Polyarteritis Nodosa.
E ANCA positive glomerulonephritis characteristically
causes nephrotic syndrome.

25 year old lady complains of unilateral throbbing


headache for 10 months. Each time she has
headaches, she also complains of unilateral
weakness in the arm which resolve
spontaneously when the headaches improve.
CT head was normal. What is the diagnosis?
A. Cluster headache
B. Migraine
C. Tension headache
D. Somatoform disorder
E. Cavernous sinus thrombosis

Answer: B) migraine. Hemiplegic migraine


is a term used to describe the migraine
syndrome that is associated with a
weakness or sensory loss of the limbs on
one side of the body. The headache
usually precedes the weakness by a day
or more. The limbs gradually return to
normal over several days.

31 yrs old female coming for infertility.


PCO by US , BMI 35. what is the most
appropriate drug?
A. Testosterone
B. clomiphene
C. Finasteride
D. Rosiglitazone
E. Metformin

Answer: clomiphene Both metformin and


clomiphene has been shown to be
effective in stimulating ovulation in
patients with PCOS (clomiphene is more
effective, but not in the options). The
reduction of hormonal imbalance and
treating insulin resistance helps to restore
the ovulatory cycles and fertility.

A 65 year old lady is seen in the rhematology


clinic with the results of a DEXA scan. The T
score of the hip is -2.8 SD and in the spine is
-2.5 SD. Which of the following medications
does NOT improve bone mineral density?
A. Calcium and vitamin D
B. Oestrogen replacement
C. Selective oestrogen receptor modulator
(SERM)
D. Bisphosphonates
E. Phosphate replacement

59- 50-year-old old man is admitted to hospital


unconsious, and smelling of alcohol. One hour
after admission, he becomes suddenly sweaty
with a regular tachycardia of 110 bpm and a
BP of 100/50. What is the diagnosis?
1) Alcohol withdrawal.
2) Hepatic encephalopathy.
3) Hypoglycaemia.
4) Subdural haematoma.
5) Wernicke's encephalopathy.

The answer is 3
This is a bit early for alcohol withdrawal
particularly as the patient is admitted
smelling of alcohol. The most likely
diagnosis is hypoglycaemia. We do not
have any clinical findings to suggest any
of the other alternativies.

A 45 year old lady has had long standing


arthritis of her hands and feet. Which of the
following X ray changes suggests rheumatoid
arthritis instead of a seronegative
arthropathy?
A. Osteosclerosis
B. Osteophytes
C. Osteoporotic changes
D. Periarticular erosions
E. Loss of joint space

Answer: d) periarticular erosions.


Osteophytes and loss of joint space are
commonly found in osteoarthritis, and
can also be found in rheumatoid arthritis.
Periarticular erosions are suggestive of
rheumatoid arthritis.

Periarticular Erosions

A 60 year old man is brought to hospital having collapsed


to the ground suddenly and was unable to move his left
leg or arm. There was no loss of consciousness. He has
a past medical history of hypertension only. The episode
lasted a few seconds and he has been relatively well.
Examination reveals a mild hemiparesis of the left arm
and leg. Which is the likely diagnosis?
A. Pontine haemorrhage
B. Primary epilepsy
C. Medullary haemorrhage
D. Right internal capsule infarct
E. Left internal capsule infarct

Answer: d) right internal capsule infarct.


This patient is likely to have a lacunar
infarct involving the internal capsule,
causing transient contralateral
hemiparesis.

A 35 year old man presents acutely with urethritis,


conjunctivitis and arthritis. Rheumatoid factor is
negative and he has raised inflammatory
markers. In view of the likely diagnosis, which
one of the following features is associated?
A. Myocardial infarction
B. B. Pulmonary embolus
C. C. Circinate balanitis
D. D. nail dystrophyE. Calcinosis

Answer: c) circinate balanitis. The diagnosis is Reiter's


syndrome. Reiters syndrome is urethritis, conjunctivitis,
seronegative arthritis (cannot see, cannot pee, cannot
climb a tree). The typical patient is a young man with
recent urethritis or dysentery. The seronegative arthritis
is usually a mono or oligoarthritis.
Other features are anterior uveitis, keratoderma
blenorrhagica (brown abscesses on palms and soles),
mouth ulcers, plantar fasciitis and archilles tendinitis
(enthesopathy), circinate balanitis (painless rash) and
aortic incompetence. The arthritis may relapse or remain
chronic. Management is usually with rest and NSAIDs.

A demyelinating polyneuropathy is typically


caused by:
1) Diabetes
2) Excessive alcohol
3) Hereditary motor-sensory neuropathy
4) Renal failure
5) Vitamin B12 deficiency

The answer is 3
The differential diagnosis of demyelinating
neuropathy includes: hereditary motor-sensory
neuropathy (Charcot-Marie Tooth disease),
Refsum's Disease, Guillain-Barre syndrome,
chronic inflammatory demyelinating
polyneuropathy (CIDP), paraprotein-related
disorder, leukodystrophies. Amiodarone,
Diabetes, alcohol, Vitamin deficiencies and renal
failure cause an axonal polyneuropathy.

A 35 year old lady has skin pigmentation,


hypotension, hyponatraemia. A short synacthen
test shows a rise in cortisol from 100 to 140
g/ml. Which one of the following conditions is
associated?
A. Papillary thyroid carcinoma
B. Ovarian fibroids
C. Phaeochromocytoma
D. Hepatocellular carcinoma
E. Pernicious anaemia

Answer: e) pernicious anaemia. Addison's


disease is described. Many autoimmune
diseases are associated e.g. vitiligo,
diabetes, primary ovarian failure and
pernicious anaemia.

A 70-year-old woman presented with episodic


impairment of consciousness.
Which of the following is the most likely cause?
1) Alzheimer type dementia
2) chronic sub-dural haematoma
3) Creutzfeldt-Jacob disease
4) depressive stupor
5) normal pressure hydrocephalus

The answer is 2
This is quite a grey question. The clinical scenario is very
brief with no mention of any neurological signs so a
logical deduction must be made.
Alzheimer's disease would be expected to have a
continous impairment of consciousness in its advanced
stages but could be episodic if there were variation in
drugs therapy or concurrent illnesses. Similarly Normal
Pressure Hydrocephalus, Creutzfeld-Jacob and
depression would present with dementia (or apparent
dementia) but not fluctuant.
Of all those listed subdural haematoma is classically
associated with fluctuating level of consciousness. This
would make it the most likely.

Temporal lobe lesions cause:


1) Apraxia
2) 2) Astereogenesis
3) Primitive reflexes
4) Visuospatial neglect
5) Wernike's (receptive) aphasia

The answer is 5
Lesions of the frontal lobe include difficulties with task sequencing and
executive skills. Expressive aphasia (receptive aphasias a temporal lobe
lesion), primitive reflexes, perseveration (repeatedly asking the same
question or performing the same task), anosmia and changes in personality.
Lesions of the parietal lobe include apraxias, neglect, astereognosis (unable
to recognise an object by feeling it) and visual field defects (typically
homonymous inferior quadrantanopia). They may also cause alcalculia
(inability to perform mental arithmetic). Lesions of the temporal lobe cause
visual field defects (typically homonymous superior quadrantanopia),
Wernike's (receptive) aphasia, auditory agnosia, and memory impairment.
Occipital lobe lesions include cortical blindness (blindness due to damage to
the visual cortex and may present as Anton syndrome where there is
blindness but the patient is unaware or denies blindness), homonymous
hemianopia, and visual agnosia (seeing but not percieving objects - it is
different to neglect since in agnosia the objects are seen and followed but
cannot be named).

A 72-year-old lady has 4 months of memory loss,


urinary incontinence and falls. On examination
she has mild memory loss and a broad-based,
slow gait. Muscle tone is normal and both
plantar reflexes are downgoing. What is the
likely diagnosis?
1) Alzheimer's disease
2) Frontal lobe dementia
3) Mulit-infarct dementia
4) Normal-pressure hydrocephalus
5) Parkinson's disease

The answer is 4
Normal pressure hydrocephalus characterized by
abnormal gait, urinary incontinence, and
dementia. It is an important clinical diagnosis,
because it is a potentially reversible cause of
dementia. It is important to distinguish it from
Parkinson's Disease. The onset of gait
disturbance and urinary symptoms is unusual so
early in dementia. Frontal lobe dementia is
characterised by loss of 'executive' functions and
multi-infarct state usually has a step-wise
history.

What is the diagnosis of the patient


whose hand on the right and suffers
hyponatremia
Slide no 10

Normal hand

A 2 year old girl with unrepaired tetralogy of Fallot presents to the


emergency department with increased cyanosis. Physical examination
shows a small, very cyanotic child lying on a stretcher with her knees drawn
up against her chest. Pulse oximetry shows an oxygen saturation of 58% in
room air. Her respirations are rapid and deep. Auscultation of the heart
discloses tachycardia but no murmur. Which of the following is the most
appropriate general treatment strategy for this patient?

Decrease systemic vascular resistance to increase right-to-left shunting


Decrease pulmonary vascular resistance to decrease left-to-right shunting
Decrease systemic vascular resistance to decrease right-to-left shunting
Increase pulmonary vascular resistance to increase left-to-right shunting
Increase systemic vascular resistance to decrease right-to-left shunting

Explanation

The correct answer is Choice E.


Understanding the anatomy of patients with congenital heart disease is crucial for understanding their care.
Often, the best way to think about the anatomy for a patient with congenital heart disease is to imagine the path of
lesion:the
pulmonary
outflow
obstruction
the VSD.
blood flow. For example,
imagine
a single
red tract
blood
cell in aand
normal
patient. The red blood cell returns to the
heart via the vena cava, deoxygenated. It then enters the right atrium, then the right ventricle, then goes to the
lungs via the pulmonary artery. After getting oxygenated in the pulmonary capillaries, it returns to the left atrium
via a pulmonary vein, then enters the left ventricle, then moves out to the body to deliver its oxygen to the tissues.
Patients with Tetralogy of Fallot have four distinct lesions ("PROVe": pulmonary outflow tract obstruction, right
ventricular hypertrophy, an overriding aorta, and a large VSD) but only two of them are necessary for
understanding the pathophysiology of the lesion: the pulmonary outflow tract obstruction and the VSD.
Imagine now a single red blood cell returning to the heart in a patient with Tetralogy of Fallot. After leaving the
vena cava and the right atrium, the RBC enters the right ventricle. Here, the cell must make a "choice" - to pass
through the stenotic pulmonary valve and enter the lungs, or to pass through the VSD and into the left ventricle
(and on to the systemic circulation) without ever getting oxygenated? Remember, blood flows to the path of least
resistance, and thus how much blood goes to the lungs versus how much goes to the body without being
oxygenated depends on the balance between the pulmonary vascular resistance and the systemic vascular
resistance.
When the pulmonary vascular resistance is high and the systemic vascular resistance is low, it is much more
difficult for blood to get into the lungs than it is for it to pass through the VSD and out to the circulation without
being oxygenated. This leads to severe right-to-left shunting, called a "Tet spell," and is what is happening to the
patient in the vignette.
Alternately, if the systemic vascular resistance is high and the pulmonary vascular resistance is low, it is easier for
blood to pass through the pulmonary outflow tract obstruction and into the lungs, where it can be oxygenated.
This is the goal of treatment for a Tet spell. Thus, the correct answer is choice E - if the systemic vascular
resistance increases, then the right-to-left shunt should decrease - which will reduce the patient's cyanosis.

Decreasing the systemic vascular resistance


would lead to increased right-to-left shunting (choice A), but this will
cause more cyanosis and would not be the goal of treatment.
Decreasing pulmonary vascular resistance (as in choice B) is a good idea, as it would allow more blood to enter
the lungs - but the goal is to decrease the right-to-left shunt, not the left-to-right.
Decreasing systemic vascular resistance (choice C) will lead to more right-to-left shunting, which is not what this
patient needs.
Increasing pulmonary vascular resistance (choice D) would cause increased right-to-left shunting, which as
described previously, would be very bad in this situation.

S-ar putea să vă placă și